Download as pdf or txt
Download as pdf or txt
You are on page 1of 81

PRE BOARDS EXAMINATION 3

NP1

Situation: A 70 year old male admitted at medical unit is recovering from Cerebrovascular
Accident (CVA) or stroke.

1. He exhibits signs of unilateral neglect. Which behavior is suggestive of unilateral neglect?


A. The client is observed shaving only one side of his face.
B. The client is unable to distinguish between two tactile stimuli presented simultaneously.
C. The client is unable to complete a range of vision without turning his head side to side.
D. The client is unable to carry out cognitive and motor activity at the same time.

2. The chart indicates that a client has expressive aphasia following a stroke. The nurse
understands that the client will have difficulty with:
A. Speaking
B. Comprehending spoken words
C. Carrying out purposeful motor activity → apraxia
D. Recognizing and using an object correctly → agnosia

Types of aphasia
● Expressive - trouble forming words
● Rexeptive - trouble understanding words

3. Which diet selection would be suited to the client? → stroke


A. Roast beef sandwich, potato chips, pickle spear, iced tea
B. Split pea soup, mashed potatoes, pudding, milk → since they have difficulty in chewing
and swallowing. Soft food and liquids are recommended
C. Tomato soup, cheese toast, Jello, coffee
D. Hamburger, baked beans, fruit cup, iced tea

4. Which nursing intervention would best improve tissue perfusion to prevent skin
problems?
A. Assessing the skin daily
B. Massaging any erythematous areas on the skin
C. Changing incontinence pads as soon as they become soiled
D. Performing range-of-motion exercises and turning and repositioning the client

5. The nurse is performing an admission history for a client recovering from a stroke.
Medication history reveals the drug clopidogrel (Plavix). Which clinical manifestation alerts
the nurse to an adverse effect of this drug? → antiplatelet
A. Epistaxis
B. Abdominal distention
C. Nausea
D. Hyperactivity

Situation: Patient Kevin, a 63-year-old with Benign Prostatic Hyperplasia (BPH), is


admitted with UTI, hematuria, and hydronephrosis. His wife says their sexual relationship
has been nonexistent due to her husband's BPH. He has used saw palmetto extract for his
urinary symptoms with some relief.

6. What is the priority nursing problem for Kevin?


A. Impaired urinary elimination
B. Social Isolation
C. Interrupted sleep pattern
D. Sexual dysfunction
7. Since the client was diagnosed with BPH, which among the following medications should
the nurse question if ordered, except?
A. Anticholinergics
B. Antihistamines
C. Decongestants
D. 5-alpha reductase inhibitor → proscar

8. Non-invasive methods were deemed ineffective for Kevin so which gold standard
procedure was prescribed to remove the enlarged portion of the prostate through an
endoscope?
A. Open prostatectomy
B. Transurethral resection of the prostate (TURP)
C. Holmium Laser Enculcatean of the Prostate (HoLEP)
D. Contact Laser Prostatectomy (CLP)

9. Suppose that Kevin had a transurethral resection of the prostate (TURP) yesterday. The
staff nurse notes that the hemoglobin is 8.2 g/dL. What is the nurse’s best action?
A. Notify the charge nurse as soon as possible.
B. Irrigate the catheter with 30 mL of normal saline.
C. Document the assessment in the medical record.
D. Prepare for a blood transfusion.

10. Kevin has used saw palmetto extract for his urinary symptoms with some relief. What
must the nurse teach the client?
A. The proper way of using saw palmetto.
B. Scientific evidence to prove they are useful is lacking.
C. It is an OTC natural substance, hence does not need consultation.
D. Use of herbs does not interfere with the action of prescribed drugs as they are natural.

Situation: Appropriate handling, treatment, and disposal of waste by type reduce costs and
do much to protect public health. The nurse correctly segregates wastes produced while
providing care to the patient.

11. Sharps from syringes should be disposed of in a puncture-proof container that is colored:
A. Yellow with a black band
B. Orange
C. Red
D. Black

Color of container/bag
Black - non-infectious dry waste
Green - non infectious wet waste (kitchen, dietary etc)
Yellow - infectious and pathological waste
Yellow with black band - chemical waste including those with heavy metal
Orange - radioactive waste
Red - sharps and pressurized containers

12. According to the DOH Health Care Waste Management Manual, the guidelines for
segregating sharps include the following, EXCEPT?
A. Sharps should be collected together, regardless of whether or not they are contaminated.
B. Containers should be puncture-proof (usually made of rubber or high-density plastic)
and fitted with covers.
C. Containers should be rigid and impermeable to contain not only the sharps but also
residual liquids from syringes.
D. Containers should be tamper-proof and needles and syringes should be rendered
unusable.

13. General practices in segregation should be observed in which among the following:
A. Accept receptacles for general waste with plastic bag cover
B. Bags and containers for infectious waste should be marked with the international
infectious substance symbol
C. Radioactive wastes should be segregated according to their physical form (solid & liquid)
and according to their half-life or potency (short-live and lived) in specially marked
containers as prescribed
D. Waste with high content of heavy metals should be collected separately and sent to waste
treatment facility available in the area

14. Chemical waste including those with heavy metals should be disposed of in a container
that is colored:
A. Yellow with a black band
B. Orange
C. Red
D. Black

15. Radioactive waste should be disposed of in a container that is colored:


A. Yellow with a black band
B. Orange
C. Red
D. Black

SITUATION: A client is admitted with a diagnosis of thrombophlebitis and deep vein


thrombosis of the right leg.

16. In the emergency room, a loading dose of heparin was administered, and IV heparin will
be administered over the next several days. The nurse should create a care plan for this
client that includes:
A. Aspirin should be taken as directed. → may inc bleeding
B. Increasing the consumption of green leafy veggies.
C. Keeping an eye on the client's prothrombin time (PT).
D. The client's activated partial thromboplastin time (aPTT) and International
Normalized Ratio (INR) are being monitored.

17. A client being discharged from the hospital for thrombophlebitis will be riding in a car
toward home. The nurse should urge the client to do the following during the 2-hour ride:
A. Carry out arm circles while riding.
B. Perform ankle pumps and range-of-motion exercises on your feet.
C. While traveling, elevate your legs.
D. Take an ambulance to your house.

Situation: Basic Life Support, or BLS, generally the type of care that first-responders,
healthcare providers provides to anyone who is experiencing cardiac arrest, respiratory
distress or an obstructed airway that requires knowledge and skills in cardiopulmonary
resuscitation (CPR), using automated external defibrillators (AED) and relieving airway
obstructions in patients of every age.

18. A client with unstable ventricular tachycardia (VT) loses consciousness and becomes
pulseless after an initial treatment with a dose of lidocaine (Xylocaine) intravenously.
Which item should the nurse caring for the client immediately obtain?
A. A pacemaker
B. A defibrillator
C. A second dose of lidocaine
D. An electrocardiogram machine

19. A client has been defibrillated at 360 joules (monophasic) and the attempts to convert
the ventricular fibrillation (VF) were unsuccessful. Based on an evaluation of the situation,
the nurse determines that which action would be best?
A. Terminating the resuscitation effort
B. Preparing for the administration of sodium bicarbonate intravenously
C. Performing cardiopulmonary resuscitation (CPR) for 5 cycles or about 2 minutes
D. Performing cardiopulmonary resuscitation (CPR) for 5 minutes, then defibrillating three
more times at 400 joules

20. The nurse is assisting to defibrillate a client in ventricular fibrillation. After placing the
pads on the client’s chest and before discharging the device, which intervention is a
priority?
A. Ensure that the client has been intubated.
B. Set the defibrillator to the “synchronize” mode.
C. Administer an amiodarone bolus intravenously.
D. Confirm that the rhythm is ventricular fibrillation.

21. Nurse Angie is working in a Surgical Unit. She receives a telephone call from the
post-anesthesia care unit stating that a client is being transferred to the surgical unit. She
plans to take which action first on arrival of the client?
A. Assess the patency of the airway.
B. Check tubes or drains for patency.
C. Check the dressing to assess for bleeding.
D. Assess the vital signs to compare with preoperative measurements.

SITUATION: Nurse bernadeth is assigned to a 50 year old, female patient with herniated
lumbar intervertebral disk who is experiencing severe pain.

22. Which position should the nurse place the client in to minimize the pain?
A. Flat with the knees raised
B. High Fowler’s position with the foot of the bed flat
C. Semi-Fowler’s position with the foot of the bed flat
D. Semi-Fowler’s position with the knees slightly raised

23. He is conducting health screening for osteoporosis. Which client is at greatest risk of
developing this problem?
A. A 25-year-old woman who runs
B. A 36-year-old man who has asthma
C. A 70-year-old man who consumes excess alcohol
D. A sedentary 65-year-old woman who smokes cigarettes

24. The nurse is visiting a home health client with osteoporosis. The client has a new
prescription for alendronate (Fosamax). Which instruction should be given to the client?
A. Rest in bed after taking the medication for at least 30 minutes.
B. Avoid rapid movements after taking the medication.
C. Take the medication with water only. → taken as soon as possible upon waking up in the
morning
D. Allow at least 1 hour between taking the medicine and taking other medication
25. Which snack selection by a client with osteoporosis indicates that the client understands
the dietary management of the disease?
A. A granola bar
B. A bran muffin
C. Yogurt
D. Raisins

26. Nurse Athena is knowledgeable about the biochemical influences of certain


neurotransmitters on ADHD. He correctly identifies biochemical influences on ADHD:
A. Norepinephrine appears to be depleted.
B. Dopamine appears to be increased. → decreased
C. Serotonin appears to be increased. → decreased
D. Oxytocin appears to be depleted.

27. The nurse identifies the priority nursing diagnosis for the patient is:
A. Risk for injury → physiological
B. Noncompliance
C. Low self-esteem
D. Impaired social interaction

28. When working with a healthcare provider to develop a treatment plan for a client, which
of the following is most likely to be included?
A. Homeschooling and anti-anxiety drugs such as buspirone (BuSpar).
B. Antidepressant drugs, such as imipramine (Tofranil), and family counseling are
recommended.
C. Monthly blood levels and anticonvulsant drugs such as carbamazepine (Tegretol).
D. Methylphenidate (Ritalin) and other stimulant drugs, as well as behavior
modification.

29. When discussing the common side effects of the medication commonly ordered for
ADHD, all of the following are correct EXCEPT:
A. Suppression of appetite
B. Insomnia
C. Growth stifling
D. Damage to the spleen

Situation: Individuals face anxiety on a daily basis. Anxiety, which provides the motivation
for achievement, is a necessary force for survival.

30. Bella knows that anxiety is correctly distinguished from fear in which statement?
A. Anxiety is cognitive process.
B. Fear is an emotional process.
C. Fear involves the intellectual appraisal of a threatening stimulus.
D. Anxiety involves the thinking response to that appraisal.

31. Nurse Bella correctly identifies the neurobiology of anxiety disorders, EXCEPT:
A. Serotonin is thought to be decreased in anxiety disorders.
B. Norepinephrine is thought to be increased in anxiety disorders
C. GABA is thought to be decreased in anxiety disorders
D. Dopamine is thought to be increased in anxiety disorders

32. Janet has a diagnosis of Generalized Anxiety Disorder. Her physician has prescribed
buspirone 15 mg daily. Janet says to the nurse, “Why do I have to take this every day? My
friend’s doctor ordered Xanax for her, and she only takes it when she is feeling
anxious.” Which of the following would be an appropriate response by the nurse?
A. “Xanax is not effective for generalized anxiety disorder.”
B. “Buspirone must be taken daily in order to be effective.”
C. “I will ask the doctor if he will change your dose of buspirone to prn so that you don’t have
to take it every day.”
D. “Your friend really should be taking the Xanax every day.”

Situation
Grief is a natural emotional response to loss that individuals must experience as they
attempt to accept the loss. The nurse is caring for various patients experiencing grief at
some points of their lives.

33. Web is currently working as a call center agent when suddenly his dog, Cappuccino,
suddenly died. He cannot take a leave which resulted short term in what type of grief?
A. Dysfunctional grief → occurs with prolonged emotional instability and a lack of
progression to successful coping with the loss
B. Disenfranchised grief → cannot be acknowledged openly (societal norms do not define
the loss as loss within its traditional definition)
C. Anticipatory grief → occurs before the loss of a loved one and is associated with an acute,
chronic, or terminal illness
D. Perceived grief

34. Grief being experienced by children is inevitable. Which among is a true response of
grief in children?
A. Toddler may see death as irreversible.
B. Child aged 6 years old has a sense of loss and is concerned about who will provide care.
C. Child aged 9 years old may begin to see death as permanent.
D. Adolescent experiencing grief do not regess.

35. The nurse’s role in the grief and loss process includes communicating with the client,
family members, and significant other. Which among the following must the nurse include in
the communication with patients, EXCEPT?
A. If you do not know what to say to a client or family who is talking about death or another
loss, listen attentively and use therapeutic communication techniques, such as open-ended
questions or reflection.
B. Realize that it is acceptable to cry with the client and family during the grief process.
C. Let the client and family know that the topic of conversation is a difficult one and that you
do not know what to say.
D. Establish trust with the client and encourage expression of feelings, concerns, and
fears within a non trusting, supportive, and nonjudgmental environment.

Situation: Patient Kenzo, a client currently admitted to the Psychiatric Ward, is diagnosed
with Schizophrenia. Nurse Frenny is caring for him.

36. Schizophrenia, as opposed to other psychotic disorders, is described?


A. Symptoms last at least 1 day but less than 1 month, and there is an eventual full return to
the premorbid level of functioning.
B. Characterized by periods of remission and exacerbation
C. The duration, including prodromal, active, and residual phases, is at least 1 month but less
than 6 months. → schizophreniform
D. With a strong element of symptomatology associated with the mood disorders
(depression or mania).
37. Currently, the most prominent neurochemical theories involve dopamine and serotonin.
Nurse Frenny, a competent nurse, knows the effects of serotonin and dopamine on
schizophrenia is? → dopamine and serotonin is decreased
A. Serotonin modulates and helps control low levels of dopamine.
B. Serotonin modulates and helps increase lowered dopamine.
C. Drugs that decrease activity in the dopaminergic system induce schizophrenic-like
reactions.
D. Drugs blocking postsynaptic dopamine receptors reduce psychotic symptoms.

38. Patient Kenzo is being given Clozapine (Clozaril), a second-generation antipsychotic.


Nurse Frenny correctly identifies the differences between typical and atypical
neuroleptics, except:
A. First-generation neuroleptics are dopamine antagonists.
B. Second-generation neuroleptics are both dopamine and serotonin antagonists.
C. Second-generation neuroleptics only diminish negative symptoms. → positive
D. First-generation neuroleptics only diminish positive symptoms.

39. One morning, while Nurse Frenny is making rounds, patient Kenzo verbalized “Someone
is trying to get a message to me through the news in this TV program; I must break the
code so that I can receive the message.” Nurse Frenny knows that the client may be
experiencing which type of delusion?
A. Persecution
B. Reference
C. Grandeur
D. Control or influence

40. The primary focus in working with the actively psychotic client in all cases would be to:
A. Promote interaction with others
B. Establish a relationship with others
C. Encourage participation in therapy activities
D. Decrease his anxiety and increase trust

Situation: Ménière’s disease is an abnormality in inner ear fluid balance caused by


malabsorption in the endolymphatic sac or a blockage in the endolymphatic duct. Nurse
Manny is caring for patient Pierre diagnosed with Meniere’s disease.

41. A glycerol test aided the diagnosis of Meniere’s disease for patient Pierre who received
an oral dose of glycerol, followed by serial audiograms over 3 hours. Which expected finding
supports the diagnosis of Meniere’s disease?
A. Improved speech discrimination
B. Impaired speech discrimination
C. Improved verbalization
D. Impaired verbalization

42. Nurse Manny attends to client Pierre who is currently experiencing vertigo. What
medication would Nurse Manny expect to be ordered for Pierre?
A. Aspirin
B. Streptomycin
C. Metoclopramide → anti-emetic
D. Furosemide
43. Nurse Manny attends to client Pierre who is currently experiencing vertigo. Banking on
the answer to the previous question, the chronic use or high doses of the dopamine
antagonist that Nurse Manny expects for patient Pierre carry the risk for?
A. Extrapyramidal Syndrome (EPS)
B. Akathisia
C. Neuroleptic Malignant Syndrome
D. Tardive dyskinesia

44. In preparing his care for patient Pierre with Meniere’s disease, nurse Manny includes the
following interventions in his nursing care plan, EXCEPT?
A. Administer IV hydration, as ordered in case of severe vomiting.
B. Keep in a bright-lit room in a comfortable position.
C. Monitor intake/output and daily weights.
D. Implement fall precautions.

45. The intratympanic injection of Gentamicin has been prescribed for patient Pierre.
Gentamicin is an antibiotic that is toxic and poisonous to the cells of the inner ear. Nurse
Manny needs to watch out for?
A. Hearing loss
B. Vertigo
C. Tinnitus
D. Aural fullness

Situation: Before partaking in their clinical duties at the psychiatric facility, nursing students
are required to participate in a set of lectures by their respective clinical instructors to gear
them up for their duties.

46. The newly arrived nursing student was asked to enumerate about the different Cluster B
Personality Disorders. She is incorrect when she includes what disorder in the list?
A. Antisocial Personality Disorder
B. Narcissistic Personality Disorder
C. Paranoid Personality Disorder → cluster A
D. Histrionic Personality Disorder

47. The clinical instructor emphasized the importance of being aware of the different
barriers in dealing with psychiatric patients. Which of the following is NOT a barrier in the
treatment of clients with personality disorders?
A. Personality characteristics and behavioral patterns are deeply ingrained
B. Course of treatment is usually quick
C. There is difficulty in modifying and changing one’s personality
D. Clients with personality disorders see their behaviors as a source of pride.

48. Personality develops through the interaction of hereditary dispositions and


environmental influences. Which of the following is NOT included in the four temperament
traits of a human being? → 4 temperament: (1) novelty seeking (2) harm avoidance (3)
reward dependence (4) perseverence
A. Harm avoidance
B. Novelty seeking
C. Intelligence quotient
D. Reward dependence
49. Although temperament is largely inherited, there are other factors that play a huge part
in influencing a person’s character. Which of the following describes the extent to which a
person considers him or herself to be an integral part of the universe?
A. Self-control
B. Self-transcendence
C. Self-directedness
D. Cooperativeness

Situation: Patient Lou, aged 68 years old, is diagnosed with Leukemia. Nurse Kia is assigned
to him.

50. Patient Lou was diagnosed with the type of leukemia that is referred to as the most
common form and most common cause of death from all leukemias. Nurse Kia knows this is?
A. Acute Lymphocytic Leukemia (ALL)
B. Chronic Lymphocytic Leukemia (CLL)
C. Acute Myelocytic Leukemia (AML)
D. Chronic Myelocytic Leukemia (CML)

51. Nurse Kia knows that the hallmark of the diagnosis of AML is?
A. Decreased erythrocytes and platelets in a complete blood count
B. Excess of blast cells (more than 20%) in a bone marrow analysis
C. Normal total leukocyte count in a complete blood count
D. Percentage of normal cells is vastly decreased

52. Nurse Kia identifies the diagnostic hallmark of CML and an important indicator of
residual disease or relapse after treatment is?
A. Montgomery chromosome
B. Juneau chromosome
C. Phoenix chromosome
D. Philadelphia chromosome

53. Overlap exists between leukemia and non-Hodgkin lymphoma (NHL) because both
involve proliferation of lymphocytes or their precursors. What distinguishes leukemia and
NHL from each other?
A. There are fewer circulating abnormal cells in lymphoma
B. There is more extensive nodal involvement in leukemia
C. There are many blast forms in the marrow in lymphoma
D. There is bone marrow involvement in NHL.

54. The management goal for patient Lou is to attain complete remission or disease
control. This means:
A. When tumor cells cannot be detected by morphologic examination but can be identified
by molecular testing.
B. There is a lack of symptoms and a normal peripheral blood smear, but there is still
evidence of disease in the bone marrow. → partial remission
C. There is no evidence of disease on physical assessment, the bone marrow and
peripheral blood appear normal, and molecular analysis shows no residual problems.
D. All molecular studies are negative for residual leukemia.

55. Nurse Kia is aware that the pathophysiology of AML is?


A. Abnormal proliferation of monoclonal immunoglobulins
B. Uncontrolled proliferation of the precursors of granulocytes
C. The proliferation of immature small B-lymphocytes
D. Excessive development of neoplastic granulocytes
Situation: Another patient of Nurse Kia is Paloma, who is diagnosed with Multiple Myeloma.

56. Nurse Kia is aware that the pathophysiology of Multiple Myeloma is?
A. Abnormal proliferation of monoclonal immunoglobulins
B. Uncontrolled proliferation of the precursors of granulocytes
C. The proliferation of immature small B-lymphocytes
D. Excessive development of neoplastic granulocytes

57. Patient Paloma complains of which among the following, as identified by Nurse Paloma
as the classic and major manifestation of Multiple Myeloma?
A. Polyuria → hypercalcemia
B. Nausea and vomiting → hypercalcemia
C. Fractures → result of bone destruction
D. Skeletal pain

58. Nurse Kia expects Zoledronic acid and Denosumab to be ordered for patient Paloma
for which purpose, EXCEPT? → inhibit bone breakdown
A. Inhibits bone breakdown
B. Inhibits bone resorption
C. Increase serum calcium
D. Alleviate skeletal pain

59. The priority nursing diagnosis for patient Paloma diagnosed with Multiple Myeloma is?
A. Impaired mobility related to bone pain and skeletal complications
B. Fatigue related to anemia, pain, and the effects of cancer treatment
C. Risk for infection related to immunosuppression and myeloma-induced immune
dysfunction
D. Risk for injury related to weakness and skeletal complications

Situation: Lucy, a 23- year old college student had been brought to the psychiatric hospital
by her parents. Her admitted diagnosis is paranoid personality disorder

60. The admitting nurse knows that which of the following CANNOT be seen in a client with a
paranoid personality disorder?
A. They maintain a considerable distance from the nurse
B. They usually choose to sit with their backs against the wall
C. They are unwary of others
D. They appear aloof and withdrawn

61. While endorsing Lucy’s case to the incoming charge nurse, the admitting nurse is also
discussing charge nurse indicates an understanding of the teaching?
A. "I should be aware that patients like Lucy really love to overreact about different things. I
should not care about any of her complaints,”
B. “Client manipulation is common in patients with personality disorders, I should make
sure to always set limits whenever dealing with them.”
C. "Restrains are always the first choice of treatment when these clients become too much to
handle,”
D. “Patients with personality disorders can be left on their own,”

62. Lucy suddenly becomes violent during lunch time. She insists that the nurses serving
their food are trying to poison and kill her. Which nursing intervention is most appropriate
to provide to her?
A. Scold Lucy because she is disturbing other patients
B. Initially restrain Lucy to promote safety
C. Use clear, calm statements and a confident physical stance.
D. Ignore Lucy and her violent behavior

63. Lucy was prescribed Risperdal. After how many days of taking the medication, the nurse
notes that Lucy became restless. Drooling is also seen and there is also weakness in her
lower extremities. Which nursing intervention would be most important to implement?
A. Document the assessment findings
B. Restrain the client
C. Give the ordered as needed dose of Cogentin → given for side effect of antipsychotics
D. Panic and inform the resident on duty

Situation: Ren, a newly hired nurse, was part of the tour of the whole hospital during the
morning shift. While on tour, the unit manager emphasized the importance of presence of
collaboration in the workplace.

64. Conflicts in the ward are common among the healthcare team members according to the
unit manager. The unit manager stated some situations including disagreement between two
employees. The manager highlighted the importance of having an arranged time agreed
mutually by the involved parties to talk about the whole situation with all the involved
members. This also pertains to as which of the following?
A. Conflict Resolution
B. Finding blame
C. Using scapegoat
D. Incident Report

65. You learned that their ward nurses utilize an approach that addresses the issue of conflict
with a resolution that satisfies both of the parties involved. Which refers to this approach
implemented by the ward?
A. Ignoring
B. Competing
C. Compromising
D. Avoiding

66. Nurse Ren stated that he wants to aid the hospital workforce through improving the
collaborative decision-making process and skills with the other healthcare professionals.
Which of the following actions will promote the collaboration between the nurses and the
interprofessional team?
A. Delaying the feedback on results
B. Excluding the patient from their decision-making process
C. Depending on the attending physician alone in making all the decision
D. Sharing responsibility of the outcome of the decision

67. A collaborative team treating a patient may include the cardiologist, social services,
dieticians, and nurse case manager as explained by the unit manager. The healthcare team,
together with the patient, works together to establish a plan of care based on the
patient’s goals. This type of collaboration would result into which of the following outcomes
for the patient?
A. Enhanced continuity of care
B. Decreased adherence with the plan of care
C. Increased healthcare costs
D. Better relationship between the physician and the patient

68. Which communication style should the nurse manager implement if she wants to
promote communication skills for a successful collaboration among the nursing team?
A. Parallel communication
B. Active listening
C. Passive communication
D. Dominance in conversation

69. After the tour, Nurse Ren is then assigned to Nurse Sasha who is a seasoned nurse for
training on a medical of the hospital. What type of nurse-to-nurse collaboration technique
demonstrates this assignment?
A. Interprofessional collaboration
B. Shared governance collaboration
C. Inter-organizational collaboration
D. Mentoring collaboration

70. During Nurse Ren’s training, he overheard a few negative comments said by his
co-nurses about other nurses on the same unit as well. He was a bit bothered upon
hearing the things they say about the others. Nurse Ren recognizes the type of behavior the
other nurses are exhibiting which is detrimental to collaboration?
A. Vertical violence → one participant has a a higher status
B. Descending violence
C. Lateral violence → one employee to another
D. Personal violence

71. Another topic emphasized by the nurse manager refers to sentinel events. Nurse Ren is
aware and comprehends that sentinel events are scenarios that refer to which of the
following?
A. An event worthy of celebrating in the unit
B. A personal milestone in the patient’s life.
C. Harms a patient by omission or commission.
D. Signals the need for immediate investigation and response.

Situation: Bernie was recently admitted to the psychiatric unit with a diagnosis of Histrionic
Personality Disorder.

72. As a nurse assigned to Bernie, which of the following is considered a characteristic


behavior of a client diagnosed with histrionic personality disorder?
A. Having odd beliefs and magical thinking → schizotypal
B. Preoccupation with perfectionism → obsessive-compulsive
C. Preoccupation with orderliness → obsessive-compulsive
D. Attention-seeking flamboyance

73. The unit manager teaches Nurse Lea about the patients diagnosed with histrionic
personality disorder and the quality of their relationships with others. Which statement
confirmed by Nurse Lea shows correct understanding about it?
A. "They are known to be strong and independent people.”
B. "Everything they do is to manipulate us all."
C. "Their interpersonal relationships tend to be shallow and fleeting, serving their
dependency needs."
D. "Their relationships with other people tend to be long lasting and genuine”

74. During the nursing process, while Nurse Lea is working with Bernie, which clinical picture
should she expect to be evident with Bernie?
A. Uses her physical appearance to gain attention.
B. Shows empathy to others.
C. Washes hand for more than three times. → obsessive-compulsice
D. Always suspicious of the nurses in the unit. → paranoid
Situation: Ryan, a twenty-three-year-old male, was sentenced for a fifteen-year sentence to
correctional unit in Manila. He was recently transferred to a psychiatric facility after not
adapting well in the correctional unit. He was diagnosed with Anti-social Personality
Disorder. → law breakers

75. The nurse-on-duty, Nurse Shey, notices that Ryan has difficulty relating to others in the
ward. Nurse Shey is aware that patients with anti-social personality disorder have this
characteristic behavior because of never having learned which of the following?
A. Being friendly
B. Being dependent on others
C. Talking with other people their age
D. Empathizing with other people

76. During the morning rounds, Ryan began acting sexually and asked Nurse Shey for her
phone number. Which of the following is the most appropriate response of Nurse Shey in
this situation?
A. “Later, when we’re all alone, I’ll give it to you.”
B. "Kiss me first and then, I’ll give it you.”
C. "I am not going to give you my number. What we have is a professional relationship
only.”
D. "I don’t usually go dating younger men.”

77. After the rejection from Nurse Shey, you learned in another assessment interview with
Ryan that he began acting out to another nurse. He started spitting, cursing, and
refusing to answer questions. What is the most appropriate statement to address this
behavior?
A. "Sir, you are being disrespectful. This is why the nurse did not give her phone number.”
B. "Stop this right now. You are starting to get on my nerves.”
C. "I understand that you are angry, but this behavior will not be tolerated."
D. “Why can’t you just stay in the correctional unit?”

78. After being reprimanded about his behavior towards the nurses, Ryan came to the
facility’s garden and decided to smoke to let off some steam. Ryan was also observed to be
lazy and was reported that he refused to do other activities and chores along with the
other patients. With the data presented, what is the primary focus of Ryan’s care plan?
A. Consistent enforcement of the unit rules and facility policy.
B. Restrain the client.
C. Let Carl do whatever he wants.
D. Making sure other clients will not follow Carl’s behavior.

Situation: Ria, the nurse-on-duty, was assigned to Cristy, a client with borderline
personality disorder.

79. Nurse Ria began to do her personal research with borderline personality disorder.
Which is a characteristic description of a pervasive mood for these clients?
A. Anisocoric
B. Dysphoric
C. Contented
D. Perfectionist

80. What is Nurse Ria’s priority nursing intervention for Cristy?


A. Maintaining consistent and realistic limits
B. Having Cristy assigned to different nurses.
C. Making sure Cristy forms a friendship with other people or patients.
D. Putting restraints from time to time.
81. During a group activity, Cristy suddenly mentions a conflict about doing personal
activities with three other nurses in the ward. Other patients in the ward began nodding their
heads and showed agreement with Cristy’s statements. They started to develop
unreasonable demands about modifying the policies. How can Nurse Ria best deal with the
situation at hand?
A. Listen to the demands of the patients and modify the policies.
B. Allow patients to modify only two of the rules they are asking for as compromise.
C. Maintain consistency of care by open communication to avoid staff manipulation.
D. Enforce lockdown as punishment of the patients’ defiance.

82. For clients diagnosed with borderline personality disorder, which nursing approach
should be used by Nurse Ria to maintain a therapeutic relationship with the client?
A. Using authoritative leadership to help clients learn to conform to society norms
B. Being firm, consistent, and empathic, while addressing specific client behavior.
C. Promoting client self-expression by implementing laissez-faire leadership
D. Overlooking inappropriate behaviors to avoid promoting secondary gains .

83. As Nurse Ria is about to leave the ward, Cristy suddenly approaches her, crying along
with blood on her wrists. She explained how Cristy cut herself due to the nurse leaving her
soon. What should Nurse Ria do?
A. Panic and immediately call the unit manager as there is injury involved.
B. Reprimand the client for inflicting injury to herself.
C. Assess the injury and the need for treatment in a calm and matter of fact manner.
D. Leave and go home since the shift is over. Let the next nurse on shift handle the crisis
since it’s their responsibility already.

84. For clients with borderline personality disorder, what technique is described in shifting
the patterns of thinking by helping clients recognize negative thoughts and feelings, and
replacing them with positive patterns of thinking?
A. Fantasy Thinking
B. Cognitive Restructuring
C. Dependability
D. Thought Stopping

85. Cristy was observed to demonstrate splitting after several days of assessment in the
unit. Nurse Ria is aware that splitting is characterized as:
A. The client is showing signs of early indication of a panic attack and a referral is necessary.
B. The client is using a defense mechanism in which all objects are seen as good or bad.
C. The client is having an intense psychotic episode.
D. The client is turning herself to another personality.

86. While Cristy and Nurse Ria are having a private nursing interaction, Cristy asks the nurse
to not divulge about her self-harm incident that she had to anyone, including the
healthcare team. What should Nurse Ria do?
A. “Okay, your secret is safe with me.”
B. “You can count on me. I dislike gossips anyway.”
C. “Why don’t you want me to tell them? I thought you wanted the concern.”
D. “Whenever something important occurs, the team needs to know about it. I will have
to tell the others, but let’s talk about it first.”

Situation: Nurse Geri is starting his first work as a novice nurse tomorrow. The day before,
he starts scanning through nursing books and recalling common terminologies to fully
prepare himself.
87. Nurse Geri is aware that the term to characterize moving a part away from the midline
is referred to as?
A. Pronation
B. Adduction
C. Rotation
D. Abduction

88. In contrast, when the nurse is moving a part towards the midline, it is referred to as:
A. Pronation
B. Adduction
C. Rotation
D. Abduction

89. It refers to the action of turning the hand in which the palm is facing posteriorly.
A. Pronation
B. Adduction
C. Rotation
D. Abduction

90. The action that involves moving a part of the body around an axis. This is referred to
as:
A. Pronation
B. Adduction
C. Rotation
D. Abduction

Situation: COVID-19 is caused by the virus known as SARS-CoV-2. As a public health nurse,
you should know the updates regarding the disease, as well as recommendations for
preventing the virus issued by different public health groups such as the CDC and WHO.

91. Melissa, a patient diagnosed with cancer who is undergoing chemotherapy, has been
found to be COVID-positive. She is observed to have fever and cough. According to the
guidelines, isolation is required for a minimum number of days starting from the
appearance of signs and symptoms which is:
A. 7 days
B. 10 days
C. 14 days
D. 21 days

92. Maddie is a fitness coach who developed a cough after having a close contact with a
gym client. The gym client, Chloe, reported to the gym that she has tested positive for
COVID-19. Maddie took a rapid antigen test which had a negative result. What is Maddie’s
next step?
A. Does not need to isolate since she is negative for COVID-19
B. Should undergo RT-PCR test.
C. Repeat rapid antigen test.
D. Proceed to the quarantine facility for further assessment.

93. In relation to the topic, the nurse can best provide protection to herself by:
A. Using alcohol or alcogel
B. Avoiding mass gatherings
C. Wearing of PPE
D. Eating meals in non-clinical area
94. The reasons for Nurse Geri to wear PPE is/are:
A. To protect the nurse
B. To protect the patient
C. A and B
D. None of the above

95. Among the following statements, which is not false regarding the COVID-19 vaccination
and the special population of pregnant clients:
A. The safety and efficacy of COVID-19 vaccination during pregnancy contribute to the
benefits outweighing any potential risks of vaccination.
B. Because pregnancies affected by COVID-19 are associated with an increased risk of
stillbirths or pre- term birth, it is recommended by the CDC that pregnant individuals receive
COVID-19 vaccination.
C. Recent studies have revealed that the antibodies produced in the pregnant individual
following vaccination are transferred to the newborn, reducing the risk of COVID-19
hospitalization in children less than 6 months of age.
D. All of the above

Situation: As development never ceases, it is important for nurses to adopt a mindset of


continuous learning, allowing them to assess, study, and adjust their clinical-approach based
on the latest evidence-based findings from systematic research in healthcare and the
nursing field.

96. The present study’s purpose is to examine the impact of perceived barriers in healthcare
delivery in relation to ethnicity, nationality, and sex on women’s early cancer detection and
women’s decision to seek care for illness symptoms. (Facione & Facione, 2007). From this
statement of purpose, we can say that the independent variable is:
A. Decisions about seeking treatment and screening
B. Women’s perceptions about barriers in the health care system
C. Women’s ethnicity, nationality, and sex
D. Women’s early cancer detection behavior

97. From the statement of purpose in question #106, the considered dependent variable is:
A. Decisions about seeking treatment and screening.
B. Women’s perceptions about barriers in the health care system.
C. Women’s ethnicity, race, gender, homosexual orientation.
D. Women’s early cancer detection behavior .

98. Obet and Tanya conducted an in-depth study about a twin and mother facing a
member’s death through suicide. Data were collected over a 1-year period through
interviews, diary notations, and conversations with all the family members. This is
characterized by which type of qualitative research?
A. Case study
B. Participatory action research
C. Narrative analysis
D. Phenomenological study

99. This refers to the process of selecting a portion of the population to represent the entire
population.
A. Sampling bias
B. Representative sample
C. Sample
D. Sampling
100. The following are under non-probability sampling except:
A. Purposive sampling
B. Convenience sampling
C. Quota sampling
D. Cluster sampling

PRE BOARDS EXAMINATION 3: NP2


Situation: Abby is a manager in a nursing unit.

1. Nurse Abby knows that the performance appraisal includes all of the following activities,
EXCEPT:
A. Sets specific standards and activities for individual performance.
B. Agency standards are used as a guide.
C. Focus activity on the correction of identified behavior/s.
D. Determine areas of strengths and weaknesses

Situation: Nurse Adele is having a history taking during an admission assessment of Holly
who is pregnant with her fourth child. Holly has a healthy 6-year-old son who was
delivered at 38 weeks and twins delivered at 28 weeks. She had 1 history of spontaneous
abortion at 19 weeks before the current pregnancy.

2. For the nursing documentation, which of the following statements below is TRUE and
CORRECT given the obstetrical history of Holly?
A. Since Holly had twins = G4P4
B. Since Holly had an abortion last year = G4P3
C. Holly is Gravida 4, Para 2 = G4P2
D. Holly is Gravida 5, Para 4 = G5P4

3. Holly is now observed to be at the second stage of labor; cervix is at 8 cm. Nurse Adele
puts her in lithotomy position, but one of the dangers of this position that should be
watched out for in Holly is _.
A. Hyperventilation
B. Hypertension
C. Hypotension
D. Tachypnea

4. If Holly experiences a difficult labor and needs to be placed in the lithotomy position for
an extended period during the second stage of labor, Nurse Adele should implement the
following precautionary measures for safety EXCEPT:
A. Use elastic bandages to be wrapped around the legs.
B. Place a small pad under her sacral area.
C. Use 2 pillows under her head.
D. Place a rolled towel under Holly’s right side of the hip.

5. Nurse Adele coaches the husband to be more supportive of Holly during the peak and
most painful phase of uterine contractions, which is called:
A. Decrescendo
B. Decrement → period of decreasing strength
C. Increment → occurs as the contraction begins in the fundus and spreads throughout the
uterus
D. Acme → period of greatest strength

6. Before the initial vaginal or internal examination, Nurse Adele must check which of the
following FIRST?
A. Holly’s blood pressure, checking for abnormalities such as hypotension or hypertension.
B. Holly’s HR, for chest compression
C. Abdominal palpation for signs of fetal distress
D. Abdominal palpation for fetal lie and position, fetal heart tone

7. In constructing a health teaching care plan, which of the following is the FIRST step that
Nurse Adele must consider knowing?
A. The client’s personal values and expectations
B. The client’s age and educational status
C. The postpartum client’s needs.
D. Certain factors that may impede the client’s learning.

8. What resources should the nurse prepare for teaching her postpartum client? Select all
that apply:
I. Pamphlets
II. Audiotapes
III. Nurse specialist
IV. Videotapes
A. 1 and 3
B. 2 and 4
C. 1, 2, and 4
D. 1, 2, 3, and 4

9. Which of the following health teaching topics should have utmost importance to be
included in the teaching of a postpartum client:
A. Personal hygiene
B. Monitor for vaginal bleeding
C. Take medications as necessary
D. Consult for reproductive health

10. Shuli went to the clinic for her first visit when she was on her 3rd month of pregnancy. Two
months later, Client Shuli came to the clinic for her second visit. She asks Nurse Bea how
much weight in pounds would she gain normally. You respond that the normal weight gain
should be:
A. 5 to 8 lbs
B. 10 to 15 lbs
C. 16 to 24 lbs
D. 24 to 32 lbs
● 1 lb per month in the trimester
● 1 lb per week in the 2nd and 3rd trimester
○ 1st - 3rd month = 3 lbs
○ 4th-5th month = 8 weeks x 1 = 8

11. Bea asks what food item she should include in her diet if the physician told her eat foods
rich in folic acid?
A. White meat
B. Oatmeal
C. Liver → has folic acid but high Vit A that can harm fetus
D. Dairy products

Situation: Mara is a newly hired nurse on night duty at the Neonatal Intensive Care Unit. She
received the endorsement from the afternoon duty that Baby Boy B and Baby Girl G are
ready for discharge. At the end of the shift, Mara gave the babies to their mothers. The day
after, Clara who was the mother of Baby Boy B came back to complain that she got the
wrong baby since she should have had a girl instead of a boy.
12. The parents are thinking about forwarding a formal complaint for gross negligence.
What are the elements that will prove that the nurse committed the complaint mentioned?
Select all that apply.
I. Nurse Mara has a duty
II. Nurse Mara failed to perform that duty
III. An injury has occurred to her patient due to that duty
IV. Causal relationship as regards the nurse to perform her duty and the resulting injury.
A. 1 and 2
B. 1, 2, and 3
C. 1, 2, and 4
D. 1, 2, 3, and 4

13. What could Nurse Mara have done to prevent this occurrence of switching of infants?
A. Checking the genitalia before giving the baby
B. Calling out the name of the mother
C. Proper identification of the infant using designated nametags
D. Identify the infant by checking their footprints.

14. Which of the following is considered the most important element that can prove that a
nurse was incompetent? The nurse was not able to perform her job ____
A. According to the set standard
B. Since she was new to the job
C. According to the orientation given
D. Because she lacks supervision

15. Should the nurse be proven guilty of incompetence after thorough investigation, what is
the primary outcome she would face as a professional nurse?
A. Delisted as a member of the APO.
B. Charged in civil court.
C. Her license to practice nursing in the country can be suspended or revoked for a
period of time by the PRC.
D. Termination from work by her employer, especially since the incident happened in the
institution.

Situation: Community Health Nursing

16. You are about to implement a health promotion intervention for one of your clients.
Which among the statements is TRUE about the focus of health promotion?
I. Enables individuals and community to increase their abilities to have control over
and improve their health.
II. Enhances the client’s physical, psychological, and spiritual well-being.
III. Engage in correcting the individual's health behavior.
IV. Prevent disease from occurring and treat it when it occurs.
A. 1 only
B. 3 and 4
C. 2 and 3
D. 1 and 2

17. In community health nursing, we know that multifactorial factors influence the health
status of the people in barangays. One of which is the health care system influences, which
of the following is NOT included?
A. Cost of healthcare services
B. Availability of technology
C. Availability of community resources and programs
D. Perception of the community on the value of available healthcare.
18. Which of the following statements pertaining to health behavior choices are important to
promote health and prevent diseases?
A. Take the right and prescribed medications at the right time and dosage.
B. Refrain from travelling to countries with a relatively high prevalence of infectious diseases.
C. Eat the correct kind of food, adequate sleep, enough physical exercise, and effective
stress management.
D. Smoking cessation and only drinking alcohol regularly.

19. As nurses, we are aware that there are behaviors that are influenced by cultural
expectations. Which statement is NOT included?
A. Drink herbal supplements to increase the immune system.
B. Have a full therapeutic body massage at least once a month.
C. Converse openly about the details of the illness.
D. Rely on information about community resources and programs.

20. Which of the following describes the important factors in the integrative models of
human health?
A. Derived solely from a physical phenomenon.
B. Generally, being attributed to psychological problems in individuals.
C. Effective only when it is combined and integrated with alternative therapies.
D. Intertwined and interactive with multiple components such as physical,
psychological, and social.

Situation: Mrs. Santos, 29 years of age, has been married for 3 years. She and her husband
have decided to postpone their pregnancy to pursue her career as a lawyer. Today, she went
to see an OB-GYNE because of vaginal spotting and to explore ways to finally get pregnant.
After diagnostic examinations, she was revealed to have endometriosis.

21. As the client asks you about the diagnosis, which of the following best gives the correct
description of endometriosis?
A. “It is caused by an infection of the endometrium.” → endometritis
B. “It is a major cause of primary dysmenorrhea.”
C. “It is caused by growth of endometrial tissues outside the uterus.”
D. “It is caused by abnormal proliferation of endometrial lining.” → endometrial hyperplasia

22. The nurse performs a physical assessment and history taking on Mrs. Santos. Which of
the following are the expected findings that are indicative of endometriosis? Select all that
apply.
I. Spotting after intercourse
II. Persistent dysmenorrhea
III. Dyspareunia
IV. Menorrhagia
A. 1, 2
B. 3, 4
C. 1, 2, 3, 4
D. 2, 3, 4

23. The patient was prescribed with Danazol (Danocrine) by the physician. The patient asks
the expected side effects again for clarification before she would go home. You know that
the patient may experience all these symptoms EXCEPT:
A. Weight gain and edema
B. Cessation of menses
C. Anovulation
D. Diminished menstrual flow
24. Included in the teaching is the client’s desire to get pregnant. You expect the physician to
discuss the possible rationale behind the client’s diagnosis interfering with fertility. Which of
the following is correct about endometriosis and possibility of infertility?
A. This is due to the pressure on the pituitary gland that consequently decreases the FSH
levels.
B. This is because of the total blockage of endometrial tissues.
C. The uterine cervix becomes inflamed and swollen causing infertility issues.
D. Due to the cease in production of adequate estrogen hormones by the ovaries.

25. At the end of the session, you as a nurse should be able to accomplish the following
EXCEPT:
A. Provide summary of notable points.
B. Make recommendations.
C. Ask if there are any queries
D. Thank the audience

Situation: It appears that Mang Ben, a 70-year-old individual, is admitted to the healthcare
facility due to malnutrition, with a weight loss of 21 lbs in recent months. He is unable to
tolerate milk and is also hypertensive, with an admitting blood pressure reading of 150/100.
Nurse Kathy has been assigned to care for him.

26. Nurse Kathy’s data that led to Mang Ben’s malnutrition are, EXCEPT:
A. On therapeutic diet
B. Edentulism → being wholly or partially toothless
C. Metabolic rate is faster with age
D. Living on his SSS pension

27. If Nurse Kathy is advising Mang Ben on protein alternatives, it's important to suggest
options that are suitable for Mang Ben's dietary needs and preferences. Here are some
protein-rich alternatives that Nurse Kathy might recommend:, EXCEPT:
A. Pork
B. Peanut butter
C. Eggs
D. Mongo

28. Which of the following interventions are suited for Mang Ben’s condition?
1. Provide mouth care when necessary to remove unpleasant tastes
2. Position Mang Ben comfortably so he can enjoy the meal more enjoyable
3. Offer a warm washcloth for hand hygiene before meals
4. Refer to dental Clinic for denture fitting
A. 1, 2, and 3
B. 1, 2, 3 and 4
C. 4 only
D. 1, 2, and 4

29. Which objective data will indicate that Mang Ben is at high risk of poor nutrition?
A. 21 lbs weight loss
B. No upper and lower masticating teeth
C. Loss of appetite
D. Lactose intolerance

30. Which of the following provides the essential nutrients to carry out the normal
physiological functioning of the body?
1. Carbohydrates
2. Protein
3. Lipids
4. Microminerals
5. Vitamins
A. 1, 2, 3 and 4
B. 1, 2 and 5
C. 1, 2, 3, and 5
D. 1, 2, 3, 4 ad 5

Situation: Nurse Kathy, a seasoned medical professional with years of experience in the
Emergency Room, immediately attends to Celia, a 35-year-old patient who has been
admitted due to severely elevated blood pressure of 190/110. Celia is also experiencing dull
headaches and dizzy spells, indicating potential complications from the high blood pressure.

31. Kathy is aware that she must also know the pulse pressure of Celia because if it is more
than 60, the patient is at risk for cardiovascular disease. Pulse pressure is the _______
A. Valuable index in determining the efficiency of heart muscles
B. Lowest degree of pressure when the heart is in its resting period just before contraction of
the left ventricle
C. Highest degree of pressure exerted by the blood pressure against the arterial wall as the
left ventricle contracts and forces the blood from it to the aorta.
D. Difference between systolic and diastolic pressure → SBP - DBP

32. Nurse Kathy needs to take the vital signs to be able to monitor the vital functions of the
body. Should she take the pulse above the lower jaw bone, the artery involved is
A. Facial
B. Radial
C. Temporal
D. Carotid

33. Nurse Kathy then takes the respiratory rate of Celia. She noticed that the respiration has
a noisy shrill and vibrating respiration. It is an abnormal respiration and is called
A. Cheyne-stokes
B. Stridor
C. Dyspnea
D. Apnea

34. During inspiration, Nurse Kathy hears a soft, low pitched, gentle sighing over the base of
the lungs. It is a normal breath sound and is referred to as _.
A. Wheezing
B. Bronchial
C. Vesicular
D. Broncho vesicular

35. After two days of confinement, Nurse Kathy observed that Celia’s breath sound has
changed and is described as fine crackling sounds. She charts this sound as _.
A. Rhonchi
B. Wheeze
C. Friction
D. Rales

Situation
In the pediatrician ward, nurse Myrna is the newest nurse. It was reinforced to her
throughout orientation that an integral part of the nurse's duties is documentation.
Protecting both the client and the nurse is a crucial part of this duty.
36. Nurse Myrna learned that the integral part of providing nursing care that serves as a
focal point for the client care assignments and reporting is _.
A. Kardex
B. Client care plans
C. Protocols
D. Nurse’s notes

37. In developing a care plan, who among the following should be involved?
1. Client
2. Family
3. Physician
4. Dietician
A. 1 and 2
B. 3 and 4
C. 2, 3, and 4
D. 1, 2, 3, and 4

38. Nurse Myrna should start her client care plan in which of the following phases of the
nursing process?
A. After assessment
B. After the formulation of the nursing diagnosis
C. Before planning
D. Before assessment

39. The Care Plan should include the following information, EXCEPT _.
A. Expected outcomes
B. Client’s needs
C. Nursing intervention
D. List of significant others

40. Nurse Myrna needs to ensure that her Client Care Plans are current and relevant. They
should be updated at least _hours?
A. 24-28
B. 24-120
C. 24-72
D. 24-36

Situation: Mylene, a registered nurse working at the Barangay Mabait Health Center, has a
vital role in promoting health and preventing illnesses within the community. Developing
health classes for primary prevention is an excellent approach to educate and empower the
residents of the barangay to maintain good health and prevent potential health issues.

41. Headnurse decided to retrain the staff nurse. She developed a training design, which is
the first item needed in writing a training design?
A. Target date
B. Training objectives
C. Financial assessment to support the training
D. Names of patients

42. Which of the following is the MOST important reminder to the staff in doing their duty in
charting the vital signs? It should be about .
A. Learning they had during the orientation
B. Policy of the hospital on charting
C. Learning they had from school
D. Policy of DOH on charting
43. The head nurse emphasized the need to plot the vital signs according to standards in
nursing service, every .
A. 3 hours
B. 2 hours
C. 4 hours
D. 1 hour

Situation: Nurse Evelyn is working toward her master's degree at one of the universities. She
wants to carry out a study project at the hospital to which she is related. Performing a study
inside the hospital where Nurse Evelyn works can offer insightful information and help
advance medical procedures.

44. Nurse Evelyn chooses a client who decided to voluntarily, freely and without coercion, to
participate in the study. Which of the following is Nurse Evelyn observing?
A. Self determination
B. Privacy and dignity
C. Fair treatment
D. Anonymity and confidentiality

45. Pediatric patients are prone to falls from hospital beds which may result in additional
cost on the part of the hospital. Which safety measures should be instituted to prevent such
incidents?
1. Restraining them
2. Having their mothers or any significant adult with them
3. Advising watchers to put up the bedrails at all times
4. Telling them they will be given injection if they will get out of bed alone
A. 3 and 4
B. 2 and 3
C. 1 and 2
D. 1 and 4

Situation: Due to a series of unexpected incidents that occurred in Municipality "A" over the
course of just 10 days, including the deaths of 2 expectant women giving birth at home, 2
children from dehydration, and 2 elderly people from pneumonia. A meeting of the staff from
the municipal health office was called by the town mayor. Corazon, a public health nurse, was
tasked with creating an action plan to stop a repeat of the incident.

46. Which of the following could have been the TOP contributors to the situation? Select all
that apply.
1. Postponement of people in seeking medical care
2. Adherence to cultural practices and beliefs
3. Lack of ambulance and drivers
4. Delay in receiving appropriate and adequate care
A. 2 and 3
B. 1 and 2
C. 1 and 4
D. 1 and 3

47. Nurse Corazon and her group prepared objectives for a plan of action. The following are
the characteristics of a good objective. Choose the BEST answer.
A. Futuristic, change-oriented, dynamic and systematic
B. Time-bound, accurate, realistic, measurable and specific
C. Flexible, accurate, top priority and feasible
D. Time bounded, measurable, change-oriented and continuous process
48. Nurse Corazon presented the plan in a community assembly for acceptance and
approval. Which of the following is the MOST important criterion for the plan’s acceptance
and approval manifested by the people.
A. Tasking the community people
B. Volunteerism and willingness of the community people
C. Presence of the municipal health officer in the assembly
D. Huge attendance in the community assembly

49. To combat the delays in attending to childbirths, Nurse Corazon adopts the Integrated
Management of Childhood Illness in the municipality. This program aims to .
A. Improve and ensure the accessibility and availability of basic and essential health
care in both public and private facilities and services
B. Ensure the quality and affordability of health goods and services
C. Develop, implement, monitor and evaluate maternal and newborn care services, ensuring
access
D. Huge attendance in the community assembly

50. In Municipality A, like any other municipalities, one MAJOR problem that makes any
health service program inaccessible is
A. Lack of community awareness
B. Limited number of health-oriented programs → availability
C. Reliance of government financial support
D. Lack of ambulance

Situation: Angelica, a 16-year-old primigravid (first-time pregnant woman) at 32 weeks


gestation, presents with a total weight gain of 20 pounds, one pound of which was gained in
the last 2 weeks. The urinalysis shows a negative result for glucose and a trace of protein.
Angelica has come to the clinic for consultation.

51. Which will BEST describe Angelica’s risk for preeclampsia?


A. Proteinuria
B. Short stature
C. Total weight gain
D. Adolescent age group

52. After instructing Angelica how to keep a record of daily fetal movement counting (DFMC)
at home, the nurse determines that the teaching has been effective when Angelica says she
will count the number of times the baby moves during which of the spans?
A. 45-minute period after lunch each day
B. 12-hour period each day
C. 30-minute period 3 times a day
D. 1-hour period each day

53. When teaching Angelica about nutritional needs, the nurse should emphasize .
A. High residue diet
B. Low sodium diet → normal sodium diet
C. Regular diet
D. High protein diet

54. The nurse mentions some of the conditions associated with preeclampsia. Which
among the feedback by Angelica to the nurse would warrant further explanation?
A. Iron deficiency
B. Physical disability
C. Multifetal pregnancy
D. Diabetes mellitus
55. Angelica was brought to the labor room for magnesium sulfate medication. The
PRIMARY purpose for its administration is to _.
A. Increase the central nervous system’s response to stimuli
B. Increase calcium absorption by the muscles
C. Decrease neuromuscular irritability
D. Reduce peripheral vascular resistance to lower blood pressure

Situation: Lydia, a G3P4 woman in her 20s, gave birth to a boy an hour ago. The newborn is
being cared for by nurse Chari.

56.Which of the following senses are already developed at birth? Select those that apply.
1. Hearing.
2. Vision.
3. Taste.
4. Smell.
A. 1, 2, 3, 4
B. 1, 3, 4
C. 1, 2
D. 1, 2, 3

57. The body system of the newborn moves through periods of irregular adjustment in the
first six (6) hours of life. It has two periods namely, the first and the second periods of
reactivity. The FIRST period of reactivity consists of the following characteristics. Select all
those that apply.
1. Alertness
2. Making such sounds
3. Slow heartbeat
4. Slow respiratory rate (RR)
A. 1, 2
B. 3, 4
C. 1 only
D. 3 only

58. Which among the following describes the Palmar Grasp Reflex of the baby? When he
A. Is startled as his crib is shaken
B. Grasps an object and placed on his toes
C. Gets an object placed on his hand by closing his fingers
D. Seems to spit out anything placed in his mouth

59. One day, Baby Boy did not want to suck from the mother’s breast. The doctor ordered
intravenous fluids. Which among these complications will Nurse Chari anticipate to occur
ALMOST IMMEDIATELY?
A. Cellulitis
B. Infiltration
C. Phlebitis
D. Burns

60. Later, the pediatrician orders Baby Boy to undergo blood transfusion because of ABO
blood incompatibility. What should be the BEST equipment to use in as much as 50mL of
blood is ordered?
A. Syringe pump
B. Soluset with microdrip
C. Pulse oximeter
D. Infusion pump
Situation: Nurse Marissa is a clinical instructor (CI) of AL University in the North. She is
assigned at the OR in a government provincial hospital. Marissa is aware of the scope of
nursing and her responsibilities to the school and the hospital.

61. What is the FIRST STEP that Marissa should do upon reporting to the OR with her
students? She should__________.
A. Orient the students to the policies of the OR
B. Introduce herself and coordinate with the head nurse
C. Greet the surgeons and anesthesiologists
D. Orient her students to the set-up of the Operating Room (OR)

62. Marissa understands her responsibilities to the school she represents and the hospital she
and her students are affiliated with. Which of the following is an INCORRECT statement
about student nurses?
A. The student nurses are expected to achieve same outcome as license nurses, that is, to do
no harm to clients
B. Although student nurses are not yet licensed they are expected to adhere to the same
standards as the licensed nurse
C. The student nurses are the sole responsibility of the CI and school.
D. Hospitals or health care agencies may impose limitations on student nurses’ practice and
coordination and collaboration by the CI and the student nurses must always be practiced.

63. When giving case assignments to student nurses at the OR, the CI should inform which of
the following OR personnel? Choose all that apply.
1. OR supervisor
2. Chief nurse
3.Scrub nurse
4. Circulator
A. 1, 2, 4
B. 3 and 4
C. 1, 3, 4
D. 1 and 2

64. If a student nurse commits a break in asepsis while an operation is on-going, who among
the following persons can call the attention of the student nurse and the entire team?
A. The CI only, in as much as she is the one directly responsible over the students.
B. The surgeon being the captain of the ship.
C. Any member of the surgical team who saw the infraction.
D. The scrub nurse or the circulating nurse on-going surgery even if she did not see the
infraction.

Situation: Every Wednesday is pre-natal day at the Hospital M Maternity Clinic, where Nurse
Mitos is assigned. She runs a program for pregnant women's health education from 8:00 to
9:00 in the morning. The topic of discussion that day was warning signs and symptoms of
pregnancy.

65. True labor is differentiated from failure labor because in true labor, contractions will _.
A. Be relieved in the side lying position
B. Bring about progressive cervical dilatation
C. Not stop when the client is encouraged to walk
D. Occur immediately after rupture of bag of water
Situation: The Community Health Nursing (CHN) service unit is the family. Olive, a public
health nurse, looks after the Puson household. The municipal building's janitor is Emanuel. A
tiny convenience shop run by his wife Lyn is located in front of their home. Their kids are
Manilyn (3 years old), Lynnel (6 years old), Emalyn (5 years old), and Lynette.

66. Based on the characteristics of the Puson family, Nurse olive can easily categorize which
of the family’s developmental stage?
A. Families with pre-school children
B. Family with school age children
C. Beginning family
D. Early child-bearing

67. In this particular stage, which of the following is a most basic concern?
A. Emergence of harmony in marital and in-law relations.
B. Child rearing
C. The couple shifts to adult social interests.
D. Balancing time and energy to meet the demands of work and family.

68. Which of the following is the BEST action that she must take?
A. Quit her job and look for another employment.
B. Disregard what she feels and continue to work independently.
C. Identify the source of conflict and understand the source of friction.
D. Seek help from the Director of Nursing.

Situation: Freda is a manager in a nursing unit.

69. As a young manager, Freda knows that conflict occurs in any organization. Which of the
following statements regarding conflict is NOT true?
A. It may result to poor performance of the staff.
B. It can be destructive, hence, it should not reach the highest level.
C. It is not beneficial, hence, it should be prevented at all times.
D. It may create a new leader from among the staff.

70. Freda tells one of the staff: “I don’t have time to discuss the matter with you now”, when
the latter asks if they can talk about an issue. Which of the conflict resolution strategies did
she use?
A. Avoidance.
B. Smoothing.
C. Compromise.
D. Restriction.

71. Freda knows that one of her staff is experiencing burnout. Which of the following is the
BEST thing to do?
A. Remind her to show loyalty to the institutions.
B. Ignore her observations; it will be resolved even without intervention.
C. Let her staff ventilate her feelings and ask how she can be of help.
D. Advise her staff to go on vacation.

Situation: Staff nurse Kyla works on the medical ward. The majority of her patients have
digestive and rectal conditions.

72. Which of the following, when assessed, separates Crohn's disease from ulcerative
colitis?
A. In Crohn's disease, bleeding is significant, whereas in Ulcerative Colitis, bleeding is less.
B. Diarrhea: In Crohn's disease, it is severe; in ulcerative colitis, it is mild
C. Affected area: Ulcerative colitis affects the ascending colon, while Crohn's disease affects
the descending colon.
D. Course of the disease: Crohn’s disease is prolonged and variable, Ulcerative colitis
has a remission and exacerbation

73. Narding exhibits signs of forgetfulness, inattention, and inappropriate responses to


some questions. The nurse predicts that in order to increase cognitive function, the
consultant will need to enlist the help of .
A. Psychologist
B. Psychiatrist
C. family Doctor
D. neurologist

Situation: The nurse must uphold the established ethical and moral standards while
practicing her job.

74. What appropriate action is the nurse expected to take if she gives the patient the
incorrect medication?
A. For the time being, keep quiet about it and pay great attention to the patient.
B. Send a report to the supervisor through the head nurse.
C. Inform the patient that a mistake has been made.
D. Ask the doctor what to do

Situation: The Psychiatric Unit employs you as a nurse. A helpful intervention in your patient
care is the application of therapeutic touch.

75. When evaluating the patient's skin turgor during a physical examination, what
therapeutic touch technique is used?
A. Friendship-warmth touch
B. Love -intimacy touch
C. Social-polite touch
D. Functional-professional

76. A patient is gently led to her room by you. This type of thing called:
A. Social-polite touch
B. Love-intimacy touch
C. Friendship-warmth touch
D. Sexual-arousal touch

77. You wrap your arms around an old patient's shoulders. What kind of contact is this?
A. Love- intimacy touch
B. Functional- professional
C. Social -polite touch
D. Friendship- warmth touch

78. What kind of contact is made when there are passionate embraces and kisses between
family members?
A. Love-intimacy touch
B. Friendship -warmth touch
C. Social -polite touch
D. Functional -professional

79. Which of the following does the nurse not utilize when caring for psychiatric patients
when using therapeutic touch?
A. Social-polite touch
B. Friendship -warmth
C. Sexual -arousal touch
D. Love -intimacy touch

Situation: Tommy, a 7-year-old first-grader, has been identified as having Attention


Deficit/Hyperactive Disorder (AD/HD).

80. What circumstance or event typically leads to the identification of ADHD symptoms?
During a child's
A. by parents at home
B. With peers, during play
C. Enrolled in the education system
D. identified at a well-baby clinic

81. Tommy frequently demonstrates the following behavioral symptoms of ADHD, with the
exception of:
A. Can't take turns and interrupt others
B. Moody and irrational behaviour
C. prone to distraction and forgetfulness
D.erroneous and sloppy work

82. Which of the following nursing diagnoses would the nurse choose for Tommy if he told
her, "I don't have friends because I'm stupid,"
A. Ineffective coping
B. Anxiety
C. withdrawal syndrome
D. low self esteem

83. Nurses may have access to extremely private patient information, so they must take all
reasonable precautions to keep it private or risk being charged of _.
A. negligence
B. malpractice
C. invasion of privacy
D. defamation

Situation: If a health institution wants to have a safe and high-quality foundation, it must
embrace quality improvement. In order to evaluate the hospital's services, the medical
director of Camiguin Medical Center visited every part of the facility five days in a row at
random.

84. Which of the following does the medical director not consider to be a quality
improvement trait?
A. The empowered one is the leader.
B. Problem-solving is by everyone
C. The employee is treated as customer
D. Response to a good or negative scenario

85. Any circumstance that could harm the hospital's reputation or its ability to provide care is
a risk. The Medical Director is aware that identifying potential risks across the institution is
necessary to avoid the following, except: (+)
A. accidents
B. injuries
C. financial loss
D. Incident reports
86. A number of concerns have been monitored using patient surveys. Which of the following
does not constitute a preventive action under the Medical Director's risk management
procedures?
A. Staff should not be defensive
B. fostering good administration-personnel relations
C. Providing safe environment every time
D. Listen to the patient’s cue carefully.

87. Which of the following drugs can be used therapeutically for inflammatory bowel
diseases?
A. Corticosteroids
B. Atropine sulfate → anticholinergic, relieves cramps
C. Dulcolax → constipation
D. Maalox → heartburn

88. Because of their rotating shifts, nurses frequently lament their lack of a personal life.
There are three main ways to find personal time, with the exception of:
A. delegate work to others
B. fill every moment with tasks or chores
C. eliminate tasks that add no value
D. hire someone else to do the work

Situation: Derek, a 72-year-old smoker and social drinker consulted the OPD due to rectal
bleeding. He was tentatively diagnosed with colorectal cancer and was advised by the
physician for admission.

89. Which of the following complaints from the client would you most likely anticipate during
your initial health history interview?
A. Change in bowel habits
B. Bouts of hematemesis
C. Projectile vomiting
D. Passing out white watery stools

90. Colonoscopy has been ordered. Which of the following actions would not be
recommended for the client?
A. Allow him to take PRN medications while taking electrolyte solution.
B. Allow him to drink the electrolyte laxatives the day before the procedure.
C. Allow him to drink chilled electrolyte solution.
D. Allow him to have liquid diet before the procedure.

91. Upon assessment, the patient’s stoma appeared dusky-purple after he underwent
colostomy. What does this indicate?
A. ischemia
B. viable stoma
C. anemia
D. necrosis

92. Why is there a need to empty the ostomy pouch before it becomes 1/3 full?
A. to prevent skin irritation and dislodging of the ostomy drainage
B. to help speed up peristalsis and feces removal
C. to prevent client from straining after surgery
D. to facilitate gas expulsion
93. After a colostomy, which of the following should the nurse IMMEDIATELY report to the
doctor?
A. Foul odor on the ostomy bag
B. Presence of pink rose to brick red stoma
C. Skin slightly detached
D. No flatus in 24 to 36 hours

Situation: The total quality management nurse informed the hospital's director that during
the last three months, both nurses and doctors had made several medication errors. They
decided to review all the cases containing these errors. Returning to the fundamentals was
strongly advised as a means of clinically improving the drug administration. The following
questions are relevant

94. Which of the following injection techniques should the nurse employ to reduce discomfort
when giving an iron drug?
1. Inject medication quickly
2. Withdraw needle quickly
3. Z-track technique
4. “Darting” needle quickly
A. 2 and 3
B. 1, 2 and 4
C. 3 and 4
D. 1, 2, 3 and 4

95. Which of the following safety measures should you take to prevent insulin injection
errors?
A. Keep the medication in the client's refrigerator and request it as needed.
B. Ask the dietary department to hold the medication until it is required.
C. Calculate the medication, then go to the patient and administer it.
D. Verify your calculations with your head nurse and take high "alert" medicines into
consideration.

96. Which of the following needle sizes should you use while administering injections to an
obese client?
A. 1-1.5 inch needle
B. 2-3 inch needle
C. 4 inch needle
D. 4 ½ inch needle

97. Some thought distortions have the potential to hinder Nurse Alex's personal and
professional growth. Which of the following statements best exemplifies the false belief that
Nurse Alex is focusing on the negatives?
A. “My nurse director is very intelligent.”
B. “I’m glad I took the job.”
C. “I always make mistakes, I’m not good enough as nurse.”
D. “I should marry Jo. She is very pretty.”

98. Nurse Alex is aware of his responsibilities and tasks as a registered nurse, particularly
with regard to attending continuing education programs. In one of the seminars he
attended, it was said that the common function for which nurses are charged of malpractice
is on
A. committing a medication error
B. failure to write an incident report
C. discharging a client who is not able to completely pay the bills
D. non-compliance to doctor’s order
99. Alex has joined the PNA as a registered nurse in the Philippines for _.
A. socialization
B. welfare and benefits
C. linkages and networking
D. personal and professional growth

100. Which of the following techniques is an effective way to prevent and control
transmission of infectious agents?
A. Hand washing
B. Alcohol-based hand rub
C. Use of mask
D.Proper gloving

PRE BOARDS EXAMINATION 3: NP3


Situation: Izzie Stevens has been admitted in the cancer center.

1. Izzie is for possible radiation therapy. Which of the following is not included as a radiation
precaution to be observed?
A. Assigns a rotating health care provider to the patient.
B. Wash gloves before removing and place in a designated container.
C. Wash hands with soap and water after removing the gloves.
D. Assign a permanent health care provider to the patient.

2. The nurse formulated a nursing diagnosis for Izzie which was risk for impaired skin
integrity with erythematous and desquamation reactions to radiation therapy. Which nursing
action is not recommended?
A. Wash the affected area with lukewarm water.
B. Wash the affected area with cold water.
C. Apply Vitamin A and D ointment to the erythematous area.
D. Advise the patient to wear cotton clothing.

3. Nurse Olivia performs her daily assessment for signs of infection in her patient, who is on
IV therapy. Which of the following interventions would help in preventing nosocomial
staphylococcal septicemia from occurring?
A. Cleanse skin with povidone iodine before venipuncture. → 70% alcohol
B. Change central venous catheter dressings every 4 hours. → 2 days
C. Change all solutions and infusion sets every 3-4 days.
D. Change peripheral short-term IV sites every other day. → 72-96 hrs

4. Nurse Olivia was receiving an endorsement from the evening shift when she noticed a
black smoke coming out of Mr. Duquette's room. Which of the following actions in the fire
safety guidelines should Nurse Olivia first implement in response to the situation?
A. Pull the nearest alarm and/or call emergency numbers
B. Rescue, protect and evacuate patient away from the fire.
C. Extinguish the fire using appropriate extinguisher.
D. Close all doors and windows to contain the fire

Situation: Ms. Grey, a widowed 68-year-old, was sent to the hospital with a complaint of a
severe headache and easy fatiguability. Her history of hypertension dates back to the age of
45. Two hours prior to admission, the patient tripped and fell in the restroom, according to
information provided by her daughter.
5. When a client has a history of falls, Nurse April should take what nursing action FIRST?
A. Complete a fall risk assessment.
B. Inspect the patient’s room
C. Conduct a focus assessment.
D. Inform her daughter about the fall risks.

Situation: An essential component of a nursing care plan is documentation. It entails


recording the nurses' assessment, nursing diagnosis, planning, interventions, and evaluation.
You have a surgical assignment, and there are patients who need your care after surgery.

6. Upon carrying a doctor's order, you have noticed that you forgot to chart the medication
given to one client. What priority nursing action would you perform?
A. Report the oversight you committed and write time order to nurse supervisor.
B. Insert information into your earlier charting and write late entry.
C. Document the information and label late entry and write time.
D. Chart in kardex and first relay to the incoming shift.

7. Nurse Amelia started documenting the nursing interventions she has done on her shift.
According to the documentation standards, which of the following is not acceptable?
A. “Ms. Shen appears dyspneic and drowsy.”
B. “The IV fluid is running based on doctor’s order.”
C. Dr. Park was informed at 6:00 AM regarding the blood pressure of Ms. Shen which 80/60.
D. Ms. Shen said “she wants to go home at 11 o’clock in the evening without doctor’s order.”

8. Lex, who was assigned to the night shift, has completed her documentation of her five
assigned patients. She now prepares to give her intershift report. Which of the following
data should be included?
1. Client’s profile
2. Nursing assessment during the shift
3. Response to medications
4. Laboratory studies and results
A. 1, 2, 3 and 4
B. 1 only
C. 2 and 3
D. 3 and 4

9. Cristina was assigned to the cardiovascular unit, where many clients are receiving therapy.
She has finished all the nursing interventions and is preparing to record the data on the
chart. Which of the following data should be included?
1. Name of the fluid ordered.
2. Time the fluid was inserted and ended
3. Who administered the fluid
4. Drops per minute required for the patient
A. 3, 4 and 5
B. 2, 3 and 5
C. 1 and 2
D. 1, 2, 3 and 4

10. Mr. Webber says to Nurse Jan, “I can’t believe that God is doing this.” Which of the
following should be Nurse Jan’s response?
A. “Is that a Bible you are reading?”
B. “It sounds that you are upset.”
C. “Would you wish to speak more to me or a chaplain about our spiritual concerns?”
D. Listen attentively and respectively to Mr. Webber
11. Nurse Nicole knows that spiritual care is needed because _.
1. it is associated with patient coping
2. patient wants it
3. it affects their decision making
4. It is associated with clinical outcome
A. 2 only
B. 1 and 4
C. 1, 3, and 4
D. 1, 2, 3, and 4

12. Nurses should be able to provide appropriate responses to which of the spiritual issues
contemplated by the dying patients?
1. “Why do people have to suffer?”
2. “Why do people live?”
3. “Is there a higher being such as God?”
4. “What does life mean?”
A. 3 only
B. 3 and 4
C. 1, 2 and 4
D. 1, 2, 3 and 4

13. Which of the behaviors depict bargaining in the five stages of dying according to
Kubler-Ross (1969)? → DABDA
A. “Lord, I will donate monthly to the Charity Ward of Hospital X if given another
chance.”
B. “Why do you have to leave us at this point of our married life?” → anger
C. “Well, life has to go on.” → acceptance
D. “Please, leave me alone. I don’t feel like talking to anyone.” → depression

14. The key activities in spiritual caring are _.


1. reading the bible for the patient
2. being with patients in their experiences during pain, sufferings and other problems
3. listening to patients when they verbally express anxieties or emotions, such as fear, anger,
loneliness, depression, or sorrow, which may be hindering the achievement of wellness.
4. touching patients either physically, emotionally, or spiritually to assure them of their
connectedness with other in the family of God
A. 1, 2 and 3
B. 1, 3 and 4
C. 1, 2, 3, and 4
D. 2, 3, and 4

Situation: Cancer patients are considered to be among the immunocompromised


population. Mary, 40 years old was diagnosed with colon cancer stage 3B.

15. To prevent her from contracting nosocomial infection, Nurse Grace would place her on ?
A. Together with normal OB patients
B. On reverse isolation
C. On isolation
D. In a general non-infectious ward

16. Nurse Grace should instruct Mary that while in the hospital that she should wear _.
A. Mask
B. Gloves
C. Foot wear
D. Apron
17. What should Nurse Grace emphasize to the family members as a top priority in order to
prevent Mary from contracting respiratory infections while she is in the hospital?
A. Stay only for 5 minutes at the room
B. Limit visits
C. Wear masks during visits
D. Refrain from bringing food from the outside

18. To prevent infection during the delivery of chemotherapy, Nurse Grace must begin the
procedure by ______.
A. Washing their hands
B. Checking the order of the physician
C. Preparing drug aseptically
D. Wearing gloves and masks

19. When Nurse Grace applies tourniquet on Mary to expose her veins for the IV
chemotherapy, she should use one that is _.
A. Clean
B. Sterile
C. Non-sterile
D. None of the above

Situation: Nurse Mariel is taking care of Angel, a 65-year-old woman who was admitted due
to cardiopulmonary disease.

20. The most effective position for clients with COPD is _______.
A. High-fowler’s
B. Dorsal
C. Dorsal recumbent
D. Sim’s

21. Which of the following is not a vital role of a nurse in caring for a COPD patient to
achieve and maintain an optimal level of health?
A. Gym enrolment for the physical exercise
B. Prescription of oxygen and medication
C. Relaxation and stress management techniques
D. Nutrition counseling

22. What is the most appropriate intervention for Mr. Santos who is refusing his medication
dose?
A. Have the patient sign a waiver that he has refused to take the medication
B. Offer option to patient to take the oral medication or medication through intravenous
infusion
C. Inform the physician and suggest to give another form of medication
D. Document the patient’s refusal to take the medication and include the reason for the
refusal

23. At endorsement time, Mr. Santos was complaining of nausea and vomiting. What is the
most appropriate action of the nurse?
A. Conduct an assessment on Mr. Santos
B. Offer a glass of ice water for oral rinse
C. Inform the Head nurse about Mr. Santos’ nausea and vomiting
D. Refer the client to the attending physician
24. The main goal of oxygen therapy for COPD patients is to prevent _.
A. Dyspepsia
B. Hypoxia
C. Hyperventilation
D. Hypoventilation

25. There are two types of oxygen delivery systems: low flow and high flow. Low flow
administration examples include all except for one.
A. Nasal cannula
B. Oxygen mask
C. Oxygen tent → >7 L
D. Oxygen hood incubator

26. The only way to ensure that patients with respiratory disease receive a correct diagnosis
is to be able to accurately collect a sputum sample, which is an important skill. Nurse Andrew
must adhere to the standards for specimen collection. The specimen collection's proper
order is .
A. Check the order, review the specimen procedure, assess patient and gather the
equipment, explain procedure to patient and perform sputum collection
B. Gather the equipment, explain procedure to patient, make patient comfortable and
perform the procedure
C. Assess the patient, explain the procedure to patient, gather equipment and perform the
procedure
D. Check the order, assess the patient, explain procedure to patient and perform sputum
collection.

27. How many sputum specimens are needed to determine the pathogen involved and to
start appropriate treatment for patients suspected of having Koch's infection? → TB
A. One
B. Two
C. Three → 8-24 hrs apart
D. Five

28. Nurse Andrew informs Jose that he will be performing the sputum specimen collection
before breakfast the next morning, as this is the best time to perform the procedure. Jose
asks why, specifically before breakfast and not any time of the day. What will be Nurse
Andrew's most appropriate response?
A. He has rested to cough out sputum
B. Lesser people are around
C. Secretions tends to accumulate during the night
D. Cooperation is better

29. If Nurse Andrew fails to obtain the minimum requirement of 15 mL of sputum, other
methods to induce sputum may be ordered, except:
A. Postural Drainage
B. Bronchoscopy
C. Use of sterile water via nebulizer
D. Nasotracheal suctioning

Situation: Mrs. Wells, a 55-year-old saleswoman, complained of swelling in her left leg for a
week. She was diagnosed with deep vein thrombosis (DVT). The nurse made his assessment
and found her left thigh 3 inches larger in circumference than the right thigh.
30. Which of the following justifies the use of intravenous treatment to administer
medication to Mrs. Wells? Select all that apply.
1. Administer non-diluted medications at a faster rate.
2. Facilitate for large volume infusion of IV fluids.
3. Volume controlled infusion
4. Pave the way for intravenous push of bolus
5. Administer diluted medications at a faster rate
A. 3, 4, and 5
B. 1, 3, and 5
C. 2, 3, and 4
D. 1, 2, and 3

31. Mrs. Wells will continue receiving heparin subcutaneously in three days to treat her deep
vein thrombosis. Mrs. Wells was administered heparin subcutaneously for the MAIN reason
that _.
A. To rapidly absorb the drug to produce faster effect
B. To provide an alternate route when drug is irritating to tissues
C. To provide a rapid long absorption of the drug to prevent pain
D. To slow down the absorption of drugs to produce a sustained effect

32. Upon performing the nursing assessment on Mrs. Wells, the salient concerns that should
be considered are the following, EXCEPT .
A. Functional impairment
B. Increase temperature of the calf or ankle
C. Anxiety related to her condition
D. Feeling of heaviness

Situation: Callie, 3 years old, is diagnosed with nephrotic syndrome and manifesting massive
proteinuria resulting to decreased of albumin in blood.

33. The nurse understand that the passage of the protein in the urine is the result of _.
A. inherited kidney disorder
B. intrinsic kidney disease
C. increased glomerular permeability
D. increased albumin production

34. Upon clinical assessment, the nurse observes that the OUTSTANDING manifestation of
the patient is
A. edema
B. weight gain
C. obesity
D. emaciation

35. What is the common physical appearance of urine in patients with nephrotic syndrome?
A. cloudy
B. frothy
C. clear
D. whitish

36. The patient with nephrotic syndrome is ordered corticosteroids. Who among the
following are not allowed in the patient’s room?
A. parents with diabetes
B. visitors with upper respiratory tract infection
C. visitors with mild asthma
D. visitors with allergy
37. The main treatments for nephrotic syndrome include corticosteroids. Which of the
following common adverse effects should nurses be on the lookout for?
A. Loss of appetite → inc
B. Loss of weight → wt gain
C. Lowering blood pressure → inc BP
D. Increase in body hair

Situation: Harry, a 26-year-old patient, is pale and complains of being easily fatigued. He
has undergone a complete blood count where abnormal cells were found. He was diagnosed
with acute lymphocytic leukemia (ALL).

38.The nurse identifies that the proper isolation precaution applied should be?
A. protective
B. standard
C. airborne precaution
D. strict isolation

39. Which of the following diagnostic procedures will definitely establish the diagnosis for
the patient Harry?
A. bone marrow biopsy
B. white blood cell count
C. complete blood count
D. hemoglobin and hematocrit counts

40. What would the physician order when thrombocyte count falls below 20,000/cu mm?
A. complete bed rest
B. strict aseptic technique
C. limit visitors
D. platelet transfusion

41. On the basis of his leukocyte count, which of the following should the nurse instruct the
patient not to do?
A. be in the private room with the doors always closed
B. limit the number of staff entering the room
C. receive immunization with live attenuated virus
D. use antimicrobial soap when bathing

42. During the period of exacerbation, the patient hemoglobin’s is markedly decreased.
Which of the following instructions is appropriate?
A. perform only activities of daily living
B. serve pork and liver barbeque
C. allow exercise as long as tolerated
D. let the patient perform self-care independently

Situation: Nurses must continually grow as a person and as a professional

43. You are expected to participate in the Continuing Professional Development program as
a newly licensed nurse working at a tertiary hospital. When the training program is the
enhancement of the competencies of nurses employed in the hospital, it is called?
A. informal training program
B. formal educational program
C. self-directed
D. in-service training program
44. The professional career development of nurse can be achieved through various ways,
such as.
A. participating in political rallies
B. attending culinary courses
C. attendance in socio-civic activities
D. engaging in CPD programs

45. What kind of response is anticipated of a concerned nurse when nurses are portrayed as
sex symbols in television advertisements?
A. condemn the issue in the radio program
B. go to the street to manifest displeasure of the nurses’ portrayal.
C. report to the concerned agency
D. keep your silence, it is the television station’s prerogative.

46. What program should a nurse attend to, if she likes to enhance her personality?
A. gymnastics
B. scuba diving
C. marathon training
D. social graces and physical fitness

47. The nurse has finally received approval for her application to move to Canada, and she
has been advised to depart in three months. She is currently enrolled in graduate school. In
addition, her mother was recently discharged from the hospital. Which of the following is
best course of action?
A. inform the agency that she can go anytime as they wish
B. inform family that the money spent in graduate school can be easily earned in Canada.
C. request the recruiter to give her more time to settle her personal concerns.
D. share with friends that this is her scape from her sad life with her family

Situation: Mike is diagnosed with stroke and suffer from a number of deficits as a result of
injury to her brain. Her rehabilitation may be long depending on the extent of brain injury.

48. Mike has been intermittently urinating in bed, which may be caused by a flaccid bladder
and difficulties with communication. He is exhibiting signs and symptoms of pressure sores as
a result of his immobility and the constantly wet bedding, which worries the nurse. In order
to best serve the patient, the nurse chooses to report this to whom?
A. attending physician
B. supervisor-in-charge
C. resident on duty
D. infectious doctor specialist

49. Mike’s stroke left him weak on the left side, causing him to become excessively quiet and
reclusive. The nurse notices that the patient appears to be quite depressed and informs the
doctor, who refers the patient to a .
A. neurologist
B. psychiatrist
C. psychologist
D. physiotherapist

50. Mike expressed concerns about continuing to be a father to his daughter after the stroke,
especially since he only had one daughter. He wants two boys but isn't confident in his
current sexual capability. Who can assist him in this area?
A. primary consultant
B. urologist
C. nurse
D. supervisor in charge

Situation: Zel, a patient diagnosed with borderline personality disorder, was admitted to
the psychiatric unit due to extreme fluctuations in mood, intense reactions, and
persistent hostility. Nurse Clark is assigned to Zel’s plan of care.

51. Considering the current condition of Zel, what is the most appropriate nursing diagnosis
that Nurse Clark should consider?
A. disturbed personal identity
B. hallucination
C. altered sensory perception
D. impaired social interaction

52. During rounds, Nurse Clark finds Zel smoking in the comfort room. Zel claims that a
housekeeping personnel gave him the lighter and matches. He says, “I did not bring these
matches with me. Scold him, not me!” Which defense mechanism is exhibited by Zel?
A. sublimation
B. reaction formation
C. denial
D. projection → blaming others

53. Zel, having been diagnosed with borderline personality disorder, must be observed
closely for which group of manifestations?
A. strong sense of self and independence
B. grief, anger and social isolation
C. altered sensory perception and thought disorder
D. clinging, acting out, mood shifts and impulsivity

Situation: Adam, a 35-year-old senior staff nurse, was diagnosed with Chronic Fatigue
Syndrome (CFS). He complains of overwhelming fatigue unrelieved by rest and having
difficulty performing activities of daily living.

54. Which of the following are the manifestations of CFS?


1. Unrefreshing sleep;
2. Decrease level of awareness;
3. Impairment in memory;
4. Muscle and joint pains;
5. Decreased muscle tone
SELECT ALL THAT APPLY.
A. 1, 3, 5
B. 1, 3, 4
C. 2, 3, 5
D. 2, 4, 5

55. Having been diagnosed with CFS, Adam’s functional abilities are deteriorating, brought
about by changes in brain chemistry. This results in a common manifestation of what
condition?
A. Decreased musculoskeletal functions
B. Depression
C. Mania
D. Decreased neuromuscular functions → basal ganglia
56. What should be the basis for the evaluation of the outcomes of care to improve
management and quality of life for Adam? → long term management
A. Absence of common complications of CFS
B. Mutual planning with the patient and family
C. Effectiveness of nursing interventions
D. Effectiveness of the medical regimen

57. Marty, a 32-year-old cancer patient who is on chemotherapy, complains of easy


fatigability, exhaustion, and diminished capacity for mental work. What is the most
appropriate nursing diagnosis for Marty?
A. Fatigue related to brain chemistry to chemotherapy
B. Fatigue related to altered body chemistry secondary to chemotherapy
C. Fatigue secondary to loss of musculoskeletal functions
D. Fatigue related to malnutrition

Situation: Nurse Tina is planning to conduct a teaching session about common types of
abuse. The following questions apply.

58. One of the most common types of abuse is child maltreatment. The following are signs of
child abuse, except?
A. “Stocking and glove” distribution marks → burns
B. Urinary Tract Infection → sexual abuse
C. Cigarettes marks
D. Belt buckle or teeth marks

59. Which of following is not a sign of sexual abuse?


A. Bruised, red, swollen genitalia
B. Skull fracture → physical abuse
C. Urinary Tract Infection
D. Rectum tear

60. The management of abuse deals with the treatment of victims and preventing repetitions
of such abuse. Which among the types of child abuse is most difficult to treat?
A. Emotional
B. Physical
C. Sexual
D. Neglect

61. Which condition may be caused by experiences of child abuse early in life?
A. Sociopath Personality
B. Narcissistic Personality
C. Hyperactive Personality
D. Dissociative Personality

62. Nurse Lily is a newly hired staff nurse. She told the head nurse that she cannot handle
one patient because the patient might be infected with HIV. Which of the following
ethical principles did Nurse Lily violate?
A. Nonmaleficence
B. Beneficence
C. Loyalty
D. Respect for human dignity
Situation: Unhealthy workplace, heavy workload, and limited resources cause stress among
healthcare professionals such as nurses. A healthy work environment plays a vital role in
nurses’ provision of quality care.

63. Burnout is becoming more prevalent among nurses. Which of the following describes
burnout?
A. Growing dissatisfaction among nurses due to changing career expectations.
B. Emotional exhaustion, depersonalization, and reduced personnel accomplishment.
C. Conflict between work life and demands of family life.
D. Increased absenteeism and rapid turnover of nurses due to a toxic work environment.

64. Physical tension can result from an unhealthy work environment. This can be managed
by progressive muscle relaxation, which involves?
A. Listening to a relaxation-audio program
B. Performing active physical exercise to unwind, like aerobic exercise
C. Focusing on an image to relax
D. Releasing muscles from tension

65. Stress management techniques are beneficial interventions for patients and nurses. The
following are part of biobehavioral stress management interventions, except?
A. Guided imagery
B. Meditation
C. Progressive muscle relaxation
D. Pharmacotherapy

66. Joe, a retired high school teacher, smokes an average of one (1) pack of cigarettes per
day. As a nurse, you educated him about the Smoking Cessation Program of DOH. Which of
the following steps does Joe need next to pursue a change in his lifestyle?
A. Commitment
B. Skills to implement change
C. Motivation
D. Information

67. To achieve an optimal level of health for Joe, lifestyle modification must be done. It
begins with: awareness, recognition
A. Attending lectures on healthy lifestyle
B. Constructive and positive attitude in life
C. Regular daily exercise
D. Recognizing the impact of unhealthy habits

Situation: One of the responsibilities of nurses working in psychiatric or mental health units
is participating in developmental research activities about mental health. The following
questions apply.

68. Nurse Shai is a beginning professional nurse working in a psychiatric unit. Considering
her level, which activity should she implement?
A. Engage in research to test knowledge and theories in nursing
B. Identify clinical problems for research study
C. Collaborate with other members of the health team to undertake research
D. Use research findings to improve clinical care
69. Nurse Shai aims to look for evidence on the efficacy of psychological interventions for
bipolar disorder. Which of the following research methodologies will give the best evidence
on this topic? → hierarchy of evidence
A. Comparative studies → 3rd
B. Meta-synthesis → multiple qualitative → 2nd
C. Meta-analysis → multiple quantitative studies → 1st
D. Critical appraisal of topic

70. Which of the following is the intervention of interest in the clinical question: “Is group
therapy more effective than cognitive-behavioral therapy in preventing relapse among
depressed patients?” → clinical question: PICO = population, intervention (next tx you want
to prove effective), comparison (existing tx, placebo), outcome (results)
A. Prevention
B. Relapse
C. Group therapy
D. Cognitive-behavioral therapy

71. Which of the following is the outcome in the research question: “Is group therapy more
effective than cognitive-behavioral therapy in preventing relapse among depressed
patients?”
A. Preventing relapse
B. Depressed patients
C. Group therapy
D. Cognitive-behavioral therapy

72. Nurse Shai aims to discover the effectiveness of psychoeducation for families of
teenagers with eating disorders. Which sampling techniques should Nurse Shai use?
A. Purposive sampling
B. Quota sampling
C. Fish-bowl technique
D. Simple random sampling

Situation: Nurse Mimi was hired as a staff nurse at the Orthopedic Ward. Being assigned to
patients with various musculoskeletal conditions, Nurse Mimi reviews herself with diagnostic
procedures and treatments commonly performed in the unit. The following questions apply.

73. The senior staff nurse asked Nurse Mimi to assess the patient’s peroneal nerve function
in relation to movement. How would Nurse Mimi perform the assessment? → mov’t and
sensation of the lower leg, foot, toes
A. Prick the skin midway between the thumb and second finger → radial nerve
B. Plantar flex toes and foot → tibial nerve
C. Dorsiflex the foot and extend the toes
D. Prick the skin midway between the great and second toe → sensation

74. The physician ordered a computed tomography (CT) for a patient with suspected
musculoskeletal condition. Which of the following is not an indication of the procedure?
A. To visualize and assess tumors
B. To identity the location and extent of fractures in areas that are difficult to evaluate
C. To assess severe trauma to the chest
D. To measure the girth of an extremity → anthropometrics: tape measure
75. Mary, a postmenopausal patient, is scheduled for a bone density scan. What should
Nurse Mimi instruct the patient to do?
A. Report any significant pain to the physician at least two days before the test. → during or
after
B. Ingest 600 mg of calcium gluconate by mouth for two weeks before the test
C. Consume foods and beverages with a high content of calcium for two days before the test.
→ false positive result
D. Remove all metal objects on the day of the scan

76. What should Nurse Mimi assess as part of history and physical examination of a patient
diagnosed with osteoarthritis? → localized joint pain
A. Anemia
B. Local joint pain
C. Weight loss
D. Osteoporosis

77. The physician ordered Ibuprofen (Motrin) for a patient with right hip pain secondary to
osteoarthritis. To decrease gastric mucosal irritation associated with the drug, Nurse Mimi
should instruct the patient to take the drug _.
A. Upon waking up
B. On an empty stomach
C. At bedtime
D. Immediately after a meal

Situation: A crisis is a turning point in an individual’s life that produces an overwhelming


emotional response. As a nurse, you encounter patients experiencing different types of crisis.
The following questions apply.

78. Coleen, a 17-year-old high school student, is admitted due to signs and symptoms of
depression one month after giving birth out of wedlock. Her mother did not accept the
baby and her boyfriend decided not to marry her. She is suffering from what type of
crisis/es?
A. Maturational (developmental crisis) and situational (unexpected events)
B. Developmental → maturational
C. Situational
D. Maturational and adventitious → social crisis; natural disasters

79. Using various therapeutic communication techniques is important in caring for patients
in crisis. The nurse tells the patient, “I’ve noticed that after you talked with your father,
you complained of a headache.” This is an example of _.
A. Reinforcement of behavior
B. Exploration of solutions
C. Suggestion
D. Clarification

80. Age-appropriate interventions are most effective to aid people in crisis return to their
previous level of functioning. For an adolescent in crisis, which among the following is most
effective?
A. Individual therapy
B. Family therapy → school aged children
C. Play therapy
D. Music therapy
Situation: Daniel, a 31-year-old truck driver, was admitted to the hospital due to abdominal
pain. Several diagnostic studies were done and he was confirmed to be diagnosed with
Pancreatitis.

81. Upon assessment, Daniel was observed to have steatorrhea. How will the nurse best
describe this?
A. Bright red stool
B. Clay-colored stool with minimal blood
C. Frothy, foul smelling stool with high fat content
D. Dark tarry stool with fatty content

Situation
As part of the Newborn Screening (NBS), Nurse Therese has been tasked by the physician to
collect blood by heel stick to an infant named Baby G born 12 hours ago. This is to assess
Baby G’s risk for congenital hypothyroidism.

82. To relieve Daniel’s abdominal pain, which of the following is the preferred medication that
produces longer action that the other drugs? → pancreatitis opioids
A. Aspirin
B. Morphine sulfate
C. Cimetidine
D. Meperidine hydrochloride

83. The nurse, together with the dietitian, is planning the prescribed diet for Daniel. Which of
the following dietary instructions is appropriate?
A. High sodium, Low carbohydrate
B. Low calorie, low carbohydrate
C. High fat, high protein
D. Low protein, high carbohydrate → low fat

84. What is the nurse’s priority health teaching for Daniel to prevent future attacks of
pancreatitis?
A. Engagement in regular exercises
B. Abstinence of alcohol intake
C. Restriction of food high in carbohydrate
D. Avoidance to stressful events

85. Carlo, a 52-year-old smoker, is diagnosed with Emphysema. Which of the following
manifestations will likely not be included in assessment findings?
A. Decreased breath sounds
B. Decreased diaphragmatic motion
C. Dullness on percussion → pneumonia, pleural effusion, tumor
D. Hyperresonance

86. Marge, a patient diagnosed with cholecystitis, is scheduled to undergo laparoscopic


cholecystectomy. What appropriate nursing diagnosis can you formulate to include in
Marge’s nursing care plan?
1. Anxiety related to surgical and environmental concerns
2. Risk for injury related to anesthesia and surgical procedures
3. Risk for pruritus related to anesthesia and surgical procedures
4. Risk for poverty related to hospital expenses
A. 1, 2, 3 and 4
B. 1 and 2
C. 1 and 3
D. 2, 3 and 4

SITUATION: As part of the Newborn Screening (NBS), nurse Therese has been tasked by the
physician to collect blood by heel stick to an infant named Baby G born 12 hours ago. This is
to assess baby G’s risk for congenital hypothyroidism.

87. Nurse Therese clarifies the order with the physician. She knows that 12 hours after birth
is not the ideal time to collect blood for which reason?
A. There is an immediate rise of thyroid stimulating hormone after birth
B. The baby needs to digest formula before a blood sample can be taken
C. A thyroid scan should be done first
D. At 24 hours, the T4 level will be extremely high

88. Baby G was diagnosed with hypothyroidism at 4 months. Nurse Therese educates the
mother about signs and symptoms consistent with the diagnosis. Select all that apply.
1. High pitched shrill cry
2. Prolonged jaundice at birth → immature livers inability to conjugate bilirubin
3. Rag-doll appearance → hypotonic muscles
4. Constipation
5. Tall for gestation at birth → short, fat, extremities
A. 1,4 and 5
B. 3, 4 and 5
C. 1, 2 and 3
D. 2, 3 and 4

89. What nursing diagnosis should Nurse Therese prioritize for Baby G? → long term mngt
A. High risk for ineffective health maintenance
B. Hyperthermia
C. Imbalanced Nutrition more than the body requirements
D. Altered oral mucous membrane

90. If congenital hypothyroidism is detected and diagnosed at a later time, irreversible


damage can occur to what major organ of the body?
A. Gastrointestinal tract
B. Liver
C. Brain → mental retardation
D. Thyroid gland

91. Lifetime management of thyroid hormone replacement is crucial to eliminate signs of


hypothyroidism. The physician prescribes Levothyroxine. Prior to administration of the drug,
what will the nurse check?
A. Blood pressure
B. Respiratory rate
C. Temperature
D. Pulse rate

Situation: Nurse Dorothea works at the Outpatient Department (OPD) of the hospital. The
healthcare services every Friday of the month are scheduled for patients with endocrine
disorders.

92. Augusta, a 32-year-old patient with suspected hypothyroidism, is admitted for further
work-up. You are assigned to develop a nursing care plan. Which of the following nursing
diagnosis is appropriate?
A. Disturbed thought processes related to hypermetabolic rate → hypometabolic rate
B. Constipation related to gastrointestinal hypermotility → hypomotility
C. Imbalanced nutrition, less body requirements → more than
D. Activity intolerance related to decreased metabolic rate

93. After diagnostic tests, Augusta is confirmed to have hypothyroidism. What possible
complication might she experience?
A. Iron and folate deficiency → anemia
B. Decreased hematocrit → anemia
C. Decreased oxygen demand → inc oxygen demand s/t atherosclerosis
D. Increased serum cholesterol

94. Betty, a patient suspected with a cardiac disorder, seeks follow-up check up. Nurse
Dorothea educates Betty the risk factors of developing complications with cardiac disorders
by stressing which risk factors to watch out for and modify?
A. Gender, stress, obesity
B. Obesity, inactivity, diet, smoking
C. Stress, family history, obesity
D. Inactivity, diet, family history

95. Nurse Dorothea recommends to patient Betty the precautionary measures to maintain a
healthy heart. Which of the following is NOT recommended?
A. Brisk walking for 15 minutes a day
B. Take 10mL of red wine every night
C. Smoking not more than a pack of cigarette a day
D. Take a diet low in saturated fats and low cholesterol

96. Patient Inez came to the hospital with a suspected eye disorder. Which of the following
characteristics should you ask as part of the assessment?
1. Is the pain sharp?
2. Is the pain burning in character?
3. Is there a history of redness?
4. Is eye discharge present?
A. 1, 2, 3 and 4
B. 1 and 2
C. 1, 3 and 4
D. 1, 2 and 3

97. Patient Inez has an initial diagnosis of retinal detachment. Which of the following
manifestations is a specific presenting complaint? → emergency but painless
A. Severe pain on the affected area
B. Excessive bleeding on the eye area
C. Sense of “floaters” in the visual field
D. Presence of excessive discharge from the eye

98. Which of the following is considered an effective method of treatment for retinal
detachment?
A. Eye implant
B. Radial keratotomy → refractive surgery to correct myopia
C. Scleral buckling
D. Use of eye drops

99. Patient Inez was advised to undergo surgery. As a nurse, what is your initial responsibility
when you prepare a patient for retinal detachment?
A. Teach family members on how to use eye drops
B. Let patient and relatives sign consent
C. Assess level of understanding about retinal detachment
D. Provide patient with proactive eyeglasses

100. Should there be no complications, the physician ordered that Patient Inez may go home
after the third postoperative day. Which of the following health instructions should you give
as the nurse?
A. Patient should initially eat small, light meals → N/V = inc IOP
B. Patient can resume her usual activities after 48 hours
C. Patient is allowed to do reading at night after discharge → limit reading
D. Patient should wear eye patch during the day

PRE BOARDS EXAMINATION 3: NP4

Situation
Mrs. Clau, a 42-year-old patient, is admitted to the facility with a chief complaint of severe
weakness after working the entire day. Assessment reveals she has been experiencing
weakness, which is more common after a full day’s work, for almost two years now. Results
of her endocrine work-up confirm the diagnosis of Grave’s disease.

1. You are the nurse in charge of Mrs. Clau. Which among the following manifestations do you
not expect to observe?
A. warm, moist skin
B. weight gain
C. bounding, rapid pulse
D. fine, silky hair

2. During endorsement, Nurse Ram told her co-nurse that Nurse Shie, a newly registered
nurse from another unit, is suffering from gonorrhea. Nurse Ram can be sued for which of
the following?
A. Defamation
B. Slander → verbal
C. Discrimination
D. Libel → written / published

3. Ms. Rafa, a Jehovah’s witness, is scheduled to undergo caesarean section. During


pre-operative teaching, she emphasized refusal to a blood transfusion intraoperatively.
Which of the following can be filled by Ms. Rafa if the nurse proceeds to transfuse blood
during the surgery?
A. Moral distress
B. Battery → unwarranted, unconsented touching
C. Trespass to person
D. Assault

Situation: Nurses have a responsibility to uphold values and ethical standards in planning
and implementing patients’ plan of care. They must closely adhere to the ethical principles
and rules and not only to the laws.

4. Nurse Lia is assigned to a patient who will undergo thyroidectomy. She ensures that the
patient has properly consented to all treatments and procedures. This reflects which
ethical principle?
A. Fidelity
B. Beneficence
C. Veracity
D. Autonomy
5. Bea, the head nurse of the Medical-Surgical ward, is planning to deploy one of her staff
nurses to the ICU due to understaffing. If the staff nurse refuses to perform duties for which
she is not qualified, she is practicing which principle?
A. Veracity
B. Beneficence
C. Respect
D. Non-maleficence

6. The Code of Ethics stipulates that human life is inviolable. Which statement correctly
translates the principle to the situation of the Filipino nurse as a professional?
A. Participating in euthanasia is allowed provided there is a doctor's order
B. Participation is permissible when the patient, family and doctor have written agreement
on it
C. After the patient, doctor and hospital administrator have agreed, nurses may be allowed
to participate
D. Nurses shall not participate in euthanasia

7. Kian, a patient diagnosed with Leukemia, is admitted to the oncology unit. When the
physician insists that Kian undergoes radiation, which is contrary to the wishes of Kian and
his family, the physician is exercising which of the following?
A. Veracity
B. Paternalism
C. Autonomy
D. Fidelity

8. Mohan, a 34-year-old Indian diagnosed with ESRD, is admitted to the hospital for kidney
transplantation. He claims his donor is a Filipino relative as required by law. Later on, it was
discovered that his claim was not true. If the doctor fails to act accordingly to the incorrect
information, the nurse must refer the case to which of the following?
A. Medical Director
B. Administrator
C. Ethics Committee
D. Chief Executive Officer

Situation: Nurse Gigi, a community health educator, conducts a teaching session on


common allergy and hypersensitivity conditions. The session is attended by adults and
parents of children with similar conditions.

9. Nurse Gigi conducts an interactive class with parents. Which communication skill, if used
by Nurse Gigi, involves active listening that is used to gain an understanding of the
attendees’ message?
A. Clarifying
B. Responding
C. Attending
D. Confronting

10. During the health education class, Nurse Gigi must be mindful of barriers in
communication so that she can _________.
A. Use them when communicating
B. Communicate better
C. Use them to enhance interactions
D. Rationalize wrong styles of communication
11. The mother of a child with atopic dermatitis named Nana is concerned about scarring
that will result from Nana’s frequent scratching. How will the nurse best communicate her
response?
A. Asking the mother to tell Nana to endure the itchiness and irritation
B. Telling the mother matter-of-fact that scars will lighten as Nana grows older anyway
C. Tapping the hand of the mother while explaining that scarring occurs only when
lesions get infected → therapeutic touch
D. Reminding the mother that beauty is only secondary to comfort derived from scratching

12. Nana’s mother tells Nurse Gigi that because of the presence of skin lesions, Nana does
not want to go out anymore and she has been telling her mother how she hates herself.
What would be Nurse Gigi’s best intervention?
A. Instructing mother to go to Nana’s primary physician
B. Giving instructions and counseling on preventive measures and treatments
C. Referring them to the dermatologist
D. Referring them to the psychologist

13. Yasmin, a 23-year-old with chronic allergic rhinitis who is taking diphenhydramine
(Benadryl), verbalized her concern about being sleepy during working hours, which may
cause her getting fired. The nurse will BEST calm Yasmin by telling her that her physician can
readily change his medicine to _.
A. Chlorpheniramine (Actifed)
B. Brompheniramine (Dimetapp)
C. Loratadine (Allerta)
D. Dimenhydrinate (Dramamine)

Situation: Knowledge of the legal framework helps nurses navigate complex ethical
dilemmas and make informed choices that uphold patient rights and professional standards.
Nurse Flores Ipsa Loquitur is currently examining the legal aspects concerning nursing
practice, particularly those related to torts.

14. What is the MOST suitable course of action for Nurse Flores after admitting a young
woman who has reported being physically assaulted and sexually abused by her
boyfriend? → medicolegal
A. Clean and bandage the client's wounds before the physician proceeds with the physical
examination.
B. Remain with the patient throughout the physical examination. → safety
C. Direct the patient to a psychiatrist for an assessment of her mental condition.
D. Advocate for the patient to contact an attorney to pursue legal action against her
boyfriend.

15. Nurse Flores knows that there is a clear delineation between the terms “assault” and
“battery”. She correctly recalls that “assault” is defined as:
A. Employing physical coercion against another person without a valid legal justification. →
battery
B. A civil wrong committed against a person or a person’s property → tort
C. Expressing intentions to cause physical harm to an individual.
D. An unlawful act committed against the community and subject to legal consequences
enforced by the state and judicial system. → crime
16. Nurse Flores admits a 10-year-old female patient. During the assessment, she notices
multiple bruises on the child and inquires with the mother about their origin. The mother
responds by stating that the child frequently experiences accidents and injures herself. In
response, the nurse should ________:
A. Avoid probing as it pertains to family affairs.
B. Suggest that the child consult with a psychiatrist.
C. Keep assessing the situation further.
D. Have a confidential conversation with the mother.

Situation: Potassium is an essential electrolyte that plays a vital role in various bodily
functions, including muscle contractions, nerve signaling, and maintaining a proper heart
rhythm. Knowing this, Nurse Confucius applies his understanding in the treatment of a
patient who has been admitted due to hypokalemia.

17. Upon Nurse Confucius' evaluation, he determines that he should concentrate on the
assessment of _______:
A. Presence of Chvostek’s sign
B. Presence of edema
C. Blood pressure
D. Heart rhythm

18. Nurse Confucius reviews the physician's order, which entails administering a 30 mEq
potassium bolus in 100 ml of normal saline over 30 minutes. His most suitable course of
action is to _______: → should not exceed 10-20 meq/hr
A. Administer the potassium following the provided instructions.
B. Seek clarification from the physician regarding the order.
C. Ask the pharmacy to provide 250 ml of saline instead of 100 ml.
D. Secure an infusion pump for potassium administration.

19. The doctor orders a thrombolytic medication for the patient. In this context, it is
understood that the intended result of this drug therapy is to ______:
A. Dissolve emboli, thereby reducing DNA damage.
B. Enhance vascular permeability and increase cerebral blood flow.
C. Reduce vascular permeability and enhance blood flow in the brain.
D. Prevent further hemorrhage within the cerebral blood vessels.

20. Nurse Koykoy documents a nursing assessment. What would be a suitable nursing
diagnosis for the young child suffering from Juvenile Rheumatoid Arthritis (JRA)? → 6 mos
until 16 yrs old
A. Impaired swallowing
B. Total Incontinence
C. Impaired Physical Mobility
D. Ineffective Airway Clearance

21. The mother inquires Nurse Koykoy about the appropriate steps to take after noticing her
child's withdrawn demeanor. Nurse Koykoy's recommendation to the mother would be to
______: → expected behavior; JRA = long term mngt
A. Advise the mother to consider arranging counseling for the child as a therapeutic
measure.
B. Offer support and empathy to her child.
C. Allocate additional time to spend with the child and less time for her other children
D. Facilitate interactions between the child and other kids with JRA.
22. Which of the following statements regarding juvenile rheumatoid arthritis (JRA) are
accurate?
1. JRA is the most common chronic condition in children under 16 years old.
2. The disease typically begins around the age of 8 and tends to peak during puberty. →
peak onset 1-3 yrs old
3. Initial symptoms are often referred to as "growing pains."
4. Children with JRA generally have a more favorable response to the medication
Methotrexate (MTX) compared to adults.
A. 1 & 2
B. 1 & 3
C. 1, 2, 3 & 4
D. 1, 3 & 4

Situation: Nurse Junas along with Nurse Juswa are both examining the results of a recent
study, which indicated that although individuals with depression may face an increased
risk of heart failure, those with cardiovascular disease are more likely to experience
symptoms of depression.

23. The study also indicates that major depression is most commonly found among which of
the following groups? → less than 40 yrs old females
A. Males
B. Females
C. Individuals over the age of 60
D. Individuals under the age of 60

24. Both Nurse Junas and Nurse Juswa have come across information suggesting that
clients with chronic heart failure experience the most significant mood disturbances
when they ________:
A. Are uncertain regarding the progression of the disease.
B. Have a history of leading highly active lives.
C. Face difficulty in maintaining employment.
D. Also have other cardiovascular disorders.

25. In the study of persistent depressive conditions, a term used to describe a state in which
the client encounters fewer than five depression symptoms lasting for a duration of two
years is referred to as .
A. minor depression → episodes of at least 2 wks of depressive sx but with fewer than 5
criteria to dx mdd
B. major depression → sx of depressed mood for at least 2 wks, interferes ADLS
C. depression psychosis → severe depression along with psychotic features like delusion,
hallucination
D. dysthymia

26. As per the research findings, the likelihood of death within a year of diagnosis in
individuals with both heart failure and major depression is _______:
A. Approximately equal to that of individuals with heart failure without major depression.
B. Twice as high as in individuals with heart failure alone.
C. Three times greater than in individuals with heart failure without major depression.
D. Tenfold compared to individuals with heart failure without major depression.

27. The study examined the factors of pharmacologic intervention in clients with both heart
failure and depression as well as psychosocial and psychotherapeutic interventions. The
study revealed that psychosocial and psychotherapeutic interventions _______:
A. Are notably less effective. → medications rather than psychosocial
B. Tend to exacerbate dyspnea.
C. May require a longer time to show effectiveness.
D. Are not favored by most clients.

Situation: Nurse Kiamoy, a nurse manager, plans to conduct a refresher training for both the
staff nurses and other healthcare team members regarding the proper procedures for
reporting and documenting any incidents that may occur within the units.

28. Nurse Kiamoy explains to the staff that incident reports serve various purposes, with the
exception of which of the following?
A. Documenting and ensuring proper follow-up.
B. Facilitating disciplinary measures.
C. Enhancing the quality of nursing services.
D. Providing an objective account of unexpected incidents.

29. Nurse Kiamoy also provided an example of a flawed reporting leading to a medication
error. In this instance, a 10-year-old child was admitted to the emergency department. One
nurse recorded the child's weight inaccurately in the chart. Subsequently, another nurse
calculated the medication dosage using this erroneous data, resulting in the child receiving
twice the required dose. The error was identified after the child had taken the medication.
What is the MOST appropriate course of action for the medication nurse to take?
A. Document the incident in an incident report.
B. Communicate with the nurse who inaccurately transcribed the weight.
C. Skip the second dose.
D. Closely monitor the child for any drug-related side effects and promptly inform the
physician. → safety

30. Nurse Kiamoy further stressed that incidents that are reported will undergo investigation
A. Promptly following the incident.
B. Within 24 hours following the incident.
C. Once the patient has been discharged from the hospital.
D. After consulting with the patient's attending physician.

31. As a nurse manager, Nurse Kiamoy also characterized the process of documenting a
client's records, which encompasses incident reports, subsequent actions, and follow-up,
as:
A. Recording
B. Implementation
C. Investigation
D. Consulting

32. The nurse manager clarifies that incidents that must be documented encompass all of the
following, EXCEPT:
A. Complaints voiced by a client or their family regarding the care they received.
B. Complaints from a staff nurse concerning a client who exhibits disruptive and
uncooperative behavior.
C. Instances where a client declines treatment.
D. Any unexpected or unexplained event that has an impact on, or the potential to impact, a
client, a family member, or staff.

33. During the nurse's assessment, the child mentions that the pain has disappeared, and
there is no longer any discomfort in his abdomen. In this situation, the nurse might consider
that the:
A. The child might be anxious about undergoing surgery.
B. The child could be struggling to communicate their pain effectively.
C. The child may be seeking additional attention.
D. The appendix may have ruptured.

34. The client's diagnosis has been verified, and the client is set to undergo an
appendectomy. Which of the following actions are included in the preoperative nursing
care?
A. Evaluate abdominal distention and listen to bowel sounds to assess bowel activity.
B. Position the child on their left side to localize and prevent the spread of infection.
C. Administer a laxative as prescribed. → inc IAP → appendix rupture
D. Apply warm compresses to the child's abdominal area to alleviate discomfort. → lead to
perforation

35. The nurse gets the child ready for surgery. What nursing actions should the nurse take
from the following options:
A. Provide clear fluids. → NPO
B. Ensure complete bed rest.
C. Apply warmth to the abdominal area.
D. Administer an enema as directed.

Source isolation = pt is infective = negative pressure room


Reverse isolation aka protective isolation = immunocompromised = positive pressure room
Strict isolation = highly infectious pts

Situation: A 70-year-old male patient arrives at the emergency department with a


complaint of being unable to move his right arm and leg, and he has no muscle tone in
that side. The initial assessment by the physician indicates a diagnosis of a left-sided
cerebrovascular accident (CVA) resulting in flaccid hemiplegia of his right side. Nurse
Chikungunya maps out a nursing care plan for the patient.

36. Nurse Chikungunya formulates a nursing diagnosis for the client. Upon her deliberation,
which of the following nursing diagnoses is the most suitable option?
A. Unilateral Neglect → 2nd priority
B. Impaired Physical Mobility
C. Sleep Pattern Disturbance
D. Activity intolerance

37. Within the first 24 hours of admission, a crucial assessment that Nurse Chiki would
conduct is an evaluation of the client's:
A. Risk factors for vascular disease
B. Health behaviors prior to admission
C. Patterns of urinary elimination
D. Size of the pupils and their response to light → identify ICP, what kind of stroke

38. Nurse Chiki conducts a functional status assessment of the client both before and after
the CVA because it is crucial for which of the following purposes?
A. Providing direction for the development of a rehabilitation plan.
B. Assisting the client in anticipating the potential recovery of their functioning.
C. Predicting the outcomes based on the client's functional status before the CVA.
D. Aiding the client in understanding their physical limitations.

Situation: Mr. Katsudon has heightened concerns about the well-being of his eyes due to a
family history of visual issues. His mother had experienced glaucoma, and his older brother
also developed this same eye condition.
Peripheral vision loss; tunnel vision → Inc IOP
39. Mr. Katsudon inquires the nurse about the most effective ways to prevent glaucoma. The
nurse's response includes which of the following points related to primary open-angle
glaucoma?
A. Utilizing protective eyewear and avoiding impacts to the head and neck.
B. Restricting exercises that raise blood pressure and avoiding sudden temperature changes.
C. Regularly conducting optic nerve assessments and tonometry tests during eye
examinations.
D. Using proper lighting and maintaining a well- balanced diet with sufficient vitamin A
intake.

40. Nurse Oyakodon has identified Mr. Katsudon as having a visual sensory and perceptual
issue related to elevated intraocular pressure. The care plan should primarily concentrate
on:
A. Managing the intense pain experienced until the optic nerve undergoes atrophy.
B. Acknowledging that damage caused by glaucoma can only be reversed in the advanced
stages of the disease. → irreversible
C. Providing guidance in advance about the potential loss of peripheral vision.
D. Promoting adherence to prescribed medication to prevent vision loss.

41. Mr. Katsudon is receiving instruction on how to apply one drop of 1% pilocarpine
hydrochloride eye drops three times daily. Nurse Oyakodon should inform him that he:
A. Needs to check his pulse before using the drops.
B. May encounter some temporary blurring of vision after applying the drops.
C. Might experience a tingling sensation in his fingers after using the drops.
D. Should keep his eyes closed for 20 minutes after applying the drops. → 2 full mins only

42. When severely high intraocular pressure occurs as a result of intravenous mannitol, the
eye doctor might contemplate treatment. Which nursing intervention should be prioritized?
→ osmotic diuretic
A. Regularly monitor blood sugar levels.
B. Place patches over both eyes.
C. Maintain well-lit conditions in the room.
D. Monitor and record all fluid intake and output. → dehydration

43. What are the fundamental elements included in a nursing model of evidence-based
practice?
A. Integration of medical and nursing approaches to deliver patient-focused care.
B. Utilization of top medical research, clinical nursing proficiency, and client preferences.
C. Incorporation of the best research evidence, clinical expertise, and the patient's
values and preferences.
D. Incorporation of clinical research, topmost medical practices, and patient's values and
preferences.

44. Which of the subsequent research categories entails the methodical gathering of
numerical data, often with substantial control?
A. Historical
B. Phenomenology
C. Qualitative → subjective
D. Quantitative

45. To comprehend how adolescents aged 16-18 perceive teenage pregnancy, which
research design would be the most suitable for addressing this research issue?
A. Grounded Theory
B. Ethnography
C. Qualitative
D. Phenomenology → perception, lived experiences, meaningful experiences

Situation: Nurse Bing Bong is currently working in the orthopedic ward in which there is a
noticeable rise in infection cases among patients in skeletal traction. Thus, the risk
management committee attempts to investigate the level of knowledge among the staff
nurses regarding this incident.

46. Which of the ensuing principles should be followed to decrease the likelihood of
infection in a patient undergoing skeletal traction? → invasive procedure
A. Initiate antibiotic treatment for the patient at least 24 hours before the procedure.
B. Administer the skeletal traction while the patient is under general anesthesia.
C. Apply skeletal traction under sterile conditions.
D. Implement counter-tractions immediately after placing skeletal pins.

47. A client in skeletal traction faces an elevated infection risk. Which of the subsequent pin
site locations are regarded as having a higher susceptibility to infection?
A. Pins inserted through the skin.
B. Pins inserted directly through the bone.
C. Areas within the line of pull.
D. Locations with substantial soft tissue. → relatively lower blood supply

48. Properly caring for pin sites is crucial nursing care for patients in skeletal traction to
minimize the infection risk. Which of the following cleansing solutions is the MOST
efficient?
A. Chlorhexidine solution → 2nd priority is normal saline
B. Soapsuds solution
C. Hydrochloric Acid Solution
D. Acetic Acid Solution

Situation: Nurse Lubby Santa is working as a staff nurse in the ENT department of a
government hospital. The department presented the results of a survey done on the
incidence of hearing loss in a barangay near an airport.

49. What would be the goal of cooperative care among the following options?
A. Addressing symptoms and mitigating the risk of hearing loss.
B. Recognizing which barangay residents require hospitalization.
C. Organizing a medical mission to provide treatment to affected barangay residents.
D. Determining which barangay residents require additional testing.

50. The multidisciplinary team engaged in the research opted to conduct additional
diagnostic examinations on specific patients to assess their hearing abilities. The
diagnostic test to be performed will be:
A. Whisper test
B. Rinne and Weber test
C. Audiometry → comprehensive hearing test
D. CT scan

Test Normal Conductibve hearing loss Sensorineural hearing loss

Rinne’s Air louder than bone Bone louder than Air Ait louder than bone
(Rinne’s +) (Rinne’s -) (rinne’s false +)

Webers Sound heard in midline Sound heard in bad ear Sound heard in good ear
51. What kind of goals would Nurse Lubby Santa establish for the barangay residents
dealing with inner ear issues among the following options? → balance and equilibrium
A. Self-care: Activities of Daily Living (ADLs)
B. Level of physical mobility
C. Respiratory status: Behavior
D. Safety measures: Preventing falls

52. What would be the primary emphasis of Nurse Lubby Santa’s health education for the
barangay residents among the following options?
A. Addressing deficits and creating coping strategies.
B. Managing nutrition
C. Maintaining hearing aid care
D. Avoiding potential complications

53. As part of preventive and promotive care functions of Nurse Lubby Santa, which of the
subsequent health-promotion practices represents the most effective approach to prevent
hearing impairment among the barangay residents?
A. Care of the ears and ear canal.
B. Advocating for environmental noise regulation and the use of ear protection.
C. Employing earplugs to safeguard the ears while swimming or driving.
D. Inspecting the hearing aid for blockages.

Situation: Mr. Mingming, a patient struggling with heroin addiction, is suspected to have
HIV-related complications. Upon recognizing the client’s potential needs and risks, a
nursing care plan for Mr. Mingming was set out.

54. The most likely reason for Mr. Mingming's HIV infection is → HIV transmission → bodily
fluids _.
A. Being exposed to contaminated bodily fluids
B. Having unprotected anal intercourse
C. His heroin addiction
D. Engaging in unprotected sexual activity with his fiancée

55. Mr. Mingming received a positive HIV test result. What explanation can you provide him
regarding this diagnosis?
A. "You have received a diagnosis of HIV, which is the precursor to AIDS."
B. "Antibodies to the HIV virus have been detected in your bloodstream."
C. "This indicates that you won't necessarily develop AIDS in the future."
D. "Currently, the AIDS virus is not actively replicating in your blood."

56. During your morning medical rounds, you observed that Mr. Papi is experiencing cough,
breathlessness, and rapid breathing. Which opportunistic infection is likely responsible for
these symptoms?
A. Cryptococcus neoformans → lungs → CNS complication
B. Toxoplasma gondii
C. Cytomegalovirus
D. Pneumocystis carinii

57. On another occasion, Mr. Papi received a visit from his girlfriend, Samara, who confessed
to having engaged in sexual activity with him. When discussing risky sexual behaviors, which
of the following will you inform Samara is not a contributing risk factor?
A. HIV can be transmitted through sexual intercourse with an infected partner.
B. Engaging in oral sex is considered to carry some risk.
C. Exposure of mucous membranes, such as getting splashes in the eyes or mouth. → oral
D. Anal intercourse is the primary mode of HIV transmission. → vaginal intercourse
58. A patient’s family member inquired about the meaning of HIV seropositivity. Your
response will be: → the person has tested positive for HIV antibodies
A. The individual tested is not currently capable of infecting others.
B. A person infected with HIV can potentially transmit the virus to their sexual partner.
C. After five years, an infected individual may be eligible to donate blood.
D. Through proper medication, the infected person may cease to be infectious within two
months.

Situation: The Quality Department has received numerous complaints, including some
related to patient falls, which prompted the department to call out agency risk management
committee.

59. A 54-year-old patient named Meiji was admitted for fever and experienced a fall from
the bed despite the presence of a caregiver. The head nurse is concerned because a fall
prevention protocol has been in place for some time. To prevent similar incidents, what is the
most suitable course of action for the head nurse to take in this situation?
A. Conduct an interview with the nurse on duty.
B. Conduct an interview with the patient.
C. Perform a root cause analysis.
D. Initiate an investigation involving all relevant parties.

60. The nurse reported to the head nurse that the side rail lock sometimes malfunctions and
may have become loose during the night, contributing to the fall incident. What is the most
suitable action for the head nurse to take in response?
A. Discipline the nurse on duty for failing to report the observation prior to the incident.
B. Issue a warning to everyone that such lapses will not be tolerated in the future.
C. Implement penalties for all nurses for not reporting the defect.
D. Draft a memorandum to the Maintenance Department requesting an inspection of
the involved bed and all other beds.

61. The quality improvement officer responsible for the units intends to hold a staff
meeting. The PRIMARY focus of the meeting is to:
A. Review the established protocol.
B. Address any identified shortcomings.
C. Reprimand those who are involved.
D. Provide information about potential consequences.

62. What is the FIRST step the nurse should take immediately after the fall to alleviate the
situation?
A. Await the arrival of the head nurse.
B. Arrange for an immediate assessment of the patient by a doctor.
C. Promptly document the incident.
D. Call the head nurse to report the incident.

63. What takeaway will the nurse gain from this incident?
A. Avoid admitting to any mistakes or oversights.
B. Ensure that her omissions go unnoticed.
C. Emphasize the importance of reporting even minor yet pertinent observations with a
focus on safety. → culture of open communication
D. Encourage others to cover her up.

Situation: Beshywap, a 64-year-old entrepreneur, was taken to the Emergency Room due to
experiencing numbness in her left face and arm, along with confusion. The ER doctor's
initial assessment indicates a possible ischemic stroke.
64. Beshywap is curious about the cause behind her ischemic stroke symptoms. This suggests
that there is:
A. A potential presence of a cerebral aneurysm. → hemorrhagic stroke
B. Spasm of cerebral blood vessels, leading to reduced blood flow to the brain. →
vaso-occlussion
C. Extravasation of blood into the brain. → hemorrhagic stroke
D. Blockage in the blood flow within cerebral blood vessels.

65. The nurse is aware that stroke can lead to visual-perceptual issues. When patient
Beshywap exhibits hemianopsia, it means she experiences:
A. Partial blindness affecting half of her visual field.
B. Difficulty comprehending spoken language. → receptive aphasia
C. Challenges in carrying out everyday movements and actions. → apraxia
D. Speech difficulties. → expressive aphasia

66. The nurse is aware that the initial diagnostic test ordered for stroke is _.
A. Carotid ultrasound
B. MRI → after CT scan; provides better detailed structures than CT scan
C. CT scan → most accessible, quickest results
D. 12-lead ECG

67. The ER nurse expects that a thrombolytic agent may be prescribed to treat ischemic
attacks. Understanding the actions of thrombolytics, the nurse should be particularly vigilant
for which adverse reaction?
A. Allergies
B. Early onset of infection
C. Formation of blood clots
D. Bleeding

68. Patient Beshywap has complained of shoulder pain. The nurse recognizes that she is
susceptible to shoulder adduction. A nursing intervention for this situation is to _______.
A. Insert one pillow in the axilla to keep the arm away from the chest.
B. Place a pillow under the arm to keep it close to the chest.
C. Position the distal joint higher than the proximal joint.
D. Position the fingers so they are barely flexed.

Situation: Cookie, a 22-year-old nurse in the general ward, is currently experiencing contact
dermatitis from gloves.

69. Individuals who develop delayed hypersensitivity to latex often report symptoms such as
_. → localized allergy
A. Rhinitis, conjunctivitis, and blisters
B. Papules, vesicles, and pruritus
C. Urticaria and laryngeal edema
D. Flushing and bronchospasm

70. The nurse understands that the diagnosis of contact latex allergy is typically based on
a patient's history and ______.
A. A skin patch test
B. The presence of IgE in serum
C. ELISA testing
D. Latex-specific IgE levels
71. Latex allergy, in its most severe form, can present as _______.
A. Blisters and other skin lesions
B. Pruritus, erythema, and swelling
C. Asthma
D. Anaphylaxis

72. Type I IgE-mediated immediate hypersensitivity reactions are promptly managed with _.
A. Epinephrine
B. Corticosteroids
C. Diphenhydramine
D. Theophylline

73. The most effective way to prevent contact latex allergy is _.


A. Quitting the job
B. Steering clear of rubberized items
C. Avoiding latex products
D. Applying lotion before putting on gloves

Situation: Nurses can make mistakes by either neglecting their responsibilities or taking
inappropriate actions while performing their duties. Therefore, they should exercise extra
caution.

74. The patient scheduled for a breast biopsy is highly anxious and appears to have difficulty
understanding her recommended procedure, which is a radical mastectomy. What is the
appropriate course of action for the nurse?
A. Provide the patient with the information that was not explained by the doctor.
B. Ask the doctor to provide the patient with more information
C. Request the available resident to provide further explanation about the surgery
D. Contact the supervisor to clarify the procedure

75. The patient declined his/her intramuscular injection, but the nurse administered it
against the patient’s wishes. What might the nurse be accused of?
A. Battery
B. Assault
C. Moral distress
D. Trespass to person

76. The staff nurse was administering prescribed modified steam inhalation to a pediatric
patient, resulting in burns. Which statement is NOT relevant in establishing negligence? →
proximal causation
A. There is a breach of the standard
B. The breach caused the harm
C. It was a verbal order from the physician
D. There is a duty of care

Situation: Patient Sally, 2 ½ years old, is brought to the ER for vomiting. According to her
mother, Sally vomited eight times in one day and passed out greenish liquid stools.
Gastroenteritis is the preliminary diagnosis.

77. You are Patient Sally’s nurse. You assess the child and observe her to be moderately
dehydrated. Your observation is based on which of the following signs?
A. Diaphoresis
B. Absence of tear formation
C. Decreased urine specific gravity → sobrang diluted
D. Vomiting
78. Patient Sally is prescribed 250 mL of intravenous fluids to run every four hours. What
rate will you set the infusion pump on? → 250 ml/ 4 hrs = 62.5
A. 10 mL/hour
B. 12 mL/hour
C. 25 mL/hour
D. 63 mL/hour

79. Before you add potassium chloride to the IV fluid, you should first ensure that Sally has
had:
A. An ECG baseline → hyperkalemia
B. Defecated
C. Voided → renal function → reduce risk of hyperkalemia
D. A laboratory test for serum calcium level

80. Patient Sally is on fluid resuscitation for hours. As her nurse, you suspect that the child
is experiencing circulatory overload when upon assessment, you noted that there is?
A. A drop in blood pressure
B. A change in respiration to slow and deep breathing
C. Moist crackles upon auscultation
D. A marked decrease in urine output

81. The doctor’s order reads that enteral feeding should be started for a patient admitted
under acute emergency care. As a knowledgeable nurse, you know that enteral feeding is
ordered _.
A. Because food may interfere with the metabolism of medications.
B. To avoid the costs of parenteral feeding.
C. When the client is able to obtain adequate nutritional requirements by eating.
D. For post-operative gastric bypass patients.

82. A contraindication for enteral feeding includes which of the following patient
characteristics?
A. Febrile
B. Agitated
C. Unable to swallow
D. With suicidal tendencies

83. While reviewing your lectures, you noted that this statement is true regarding nasogastric
tubes. “Nasogastric tubes…”
A. “Do not require pumps for continuous infusions”.
B. “Are used for short-term therapy”.
C. “Should be inserted by a physician”.
D. “Contain eyelets along the tube which help in controlling flow and reducing clogging”.

84. Proper insertion of a nasogastric tube is needed for patient safety. Which landmarks are
involved as markers in inserting NG tubes?
A. Sternal notch, nasal bridge, and xiphoid process
B. Xiphoid process, umbilicus, and tip of nose
C. Top of forehead, nares, and sternal notch
D. Tip of nose, tip of ear, and xiphoid process

85. Nurse Pia is aware of her responsibilities when giving medications to a patient with
enteral feeding. Which of the following option(s) is/are correct?
A. Coordinate with the pharmacist regarding the timing of medications.
B. Dilute liquid medications with water.
C. Do not mix feedings and medications.
D. All of the above.

Situation: Mr. Santos, a 45-year-old male, is admitted to the oncology unit for prostate
cancer. Nurse Kate interacts with the patient who is scheduled for prostatectomy.

86. Mr. Santos says, “I feel nervous thinking about the operation I am going to have
tomorrow”. As the nurse, what is your best response?
A. “I will ask your doctor for some medication to help you relax”.
B. “Before admission, you should have asked other patients who underwent prostatectomy
about what they felt and experienced”.
C. “Doctor Ausquin is the best surgeon in this hospital, so you are in good hands!”
D. “It is alright to feel nervous. I don’t recall anyone who was not nervous before an
operation”.

87. Mr. Santos becomes irritated and agitated, telling Nurse Kate, “I am now unsure that I
want to be operated on!” Nurse Kate’s most therapeutic response is:
A. “You seem upset about your upcoming operation”.
B. “You don’t want to have the operation now?”
C. “I will call your doctor so he can help you”.
D.“Do you want me to call on the doctor so he can postpone the operation for now?”

88. Mr. Santos continues, saying, “I don’t know if I can survive the operation”. Nurse Kate
would not want to reply which?
A. “You fear that you’ll die?”
B. “Don’t worry, everything will be okay”.
C. “Undergoing an operation can be a frightening experience”.
D. “You sound afraid”.

89. Mr. Santos says to Nurse Kate, “My wife is quite upset because she thinks that the
surgery will make me impotent”. Nurse Kate’s best response should be:
A. “I am positive that your wife loves you enough to make this sacrifice, especially if it’s for
your own health and well-being”.
B. “The surgeon will be very careful and precise during the operation”.
C. “Your wife may not be as keen about sex than you think she is”.
D. “Let’s talk about how you feel regarding the operation”.

90. “I think I am going to die on the operating table.” To which, Nurse Kate responds, “You
feel as though you are going to die?” Which therapeutic communication technique did
Nurse Kate utilize?
A. Validating / restating
B. Reflecting
C. Paraphrasing
D. Focusing

Situation: Nurse Thalia is working in the emergency room. A 22-year-old vehicular accident
victim is brought to the ER with apparent head injury. He is unconscious and exhibiting
signs of increasing ICP.

91. When the patient arrives in the ER, the initial priority is?
A. Stopping bleeding from the open head wound.
B. Replacing blood loss.
C. Establishing an airway.
D. Assessing if there is fracture in the cervical area.
92. The most appropriate position for this patient is?
A. Left Sim’s
B. Trendelenburg
C. Elevate head using two pillows
D. Elevate head of bed to 30-45 degrees

93. Nurse Thalia should be aware of the signs of increasing ICP, which includes?
A. Unequal pupil size
B. Tachycardia
C. Decreasing body temperature
D. Decreasing systolic pressure

94. Nurse Thalia is monitoring the patient. She alerts the team when which respiratory sign is
assessed, indicating that there is increasing intracranial pressure in the brainstem?
A. Nasal flaring
B. Asymmetric chest excursion
C. Rapid, shallow respirations
D. Slow, irregular respirations

95. Nurse Thalia should be alert for the earliest indication of neurologic deterioration in the
client, which is?
A. Fixed, dilated pupils
B. Widening pulse pressure
C. Bradycardia
D. A change in level of consciousness

Situation: Nurse Pia from the oncology unit is caring for Patient Elle, 35-years-old, female,
diagnosed with colon cancer.

96. Elle tells Nurse Pia that she has a family history of colon cancer. Which is the most
appropriate action by Nurse Pia?
A. Inform the patient to ask her physician about diagnostic tests that specify colon cancer.
B. Educate the patient that she needs to undergo a colonoscopy once she reaches 50 years
old.
C. Instruct the patient step-by-step on how to get a stool specimen for occult blood.
D. Tell the patient to inform her physician of her need for a sigmoidoscopy which helps
provide baseline data for patients with colon cancer.

97. Patient Elle is scheduled for external radiation therapy to the abdomen. Which
information below warrants further information by the nurse-in-charge? The patient _.
A. Swims in chlorinated pools five days a week. → drying, irritation → lesions
B. Showers with a moisturizing soap everyday.
C. Eats a small amount of food five to six times during the day.
D. Has a history of dental caries.

98. The physician comes in and tells the patient, “The tumor cells in the bowels are poorly
differentiated”. If Elle asks Nurse Pia what this means, Nurse Pia knows that:
A. “The tumor cells have DNA that is different from your normal bowel cells”.
B. “These are immature fetal cells which look different from normal bowel cells”.
C. “The cells in the tumor have mutated from normal bowel cells”.
D. “The cells in your tumor do not look any different from normal bowel cells”.
99. Patient Elle appears irritated, anxious, and angry, leading Nurse Pia to the nursing
diagnosis, “Anxiety related to fear of the unknown, as manifested by anger”. To provide
more information related to the patient’s medical diagnosis, the most appropriate action to
be done for the patient is?
A. Provide suggestions to modify the patient’s expression of anger.
B. List a detailed plan for future interventions.
C. Simply explain the proposed treatments to the patient.
D. Educate the patient on specific yet simple scientific facts related to colon cancer.

100. As a knowledgeable nurse, you are aware that the legal terminology used in child abuse
is “battery”. Battery pertains to:
A. Performing an act that a prudent person with the same education or experience would not
do.
B. A legal wrong done by one person against the property of another.
C. Threatening the character of a person while attempting to do bodily harm.
D. Applying brute force to another person without lawful justification.

PRE BOARDS EXAMINATION 3: NP5


Situation: Patient X, 70 years old, male, is brought to the ER due to complaints of (1)
inability to move his right arm and leg and (2) absence of muscle tone. The presenting
diagnosis is left cerebrovascular accident (CVA) with flaccid hemiplegia of his right side.

1. You, as the nurse, are thinking of a nursing diagnosis for the client. Which of the following
nursing diagnoses is most appropriate?
A. Impaired physical mobility
B. Unilateral neglect
C. Activity intolerance
D. Sleep pattern disturbance

2. During the first 24 hours of admission, a priority assessment that you should consider is?
A. Health behaviors prior to the injury
B. Urinary elimination patterns
C. Risk assessment for vascular disease
D. Pupil size and pupillary response

3. Performing a functional status assessment before and after the CVA is crucial for what
reason?
A. It helps the client internalize his physical limitations.
B. Functional status prior to the insult helps assess prognosis.
C. It helps the patient expect to regain much of his abilities.
D. It serves as a guide for the rehabilitation plan.

4. The physician-in-charge orders potassium via IV. Which of the following nursing actions
warrants immediate attention?
A. Agitating the bag prior to hooking it up to prevent bolus administration.
B. Diluting the potassium in normal saline.
C. Administering the potassium via IV push.
D. Changing the IV insertion site every 72 hours or sooner if there is phlebitis.

5. You observe that while treating a patient with hypokalemia, none of the interventions to
raise the potassium level are working. Which pertinent patient assessment is significant to
this situation?
A. The patient has chronic insomnia.
B. The patient has a recent severe leg fracture.
C. The patient uses estrogen.
D. The patient abuses laxatives.

6. If a patient has hypokalemia, which of the following food choices is the best potassium
source?
A. White rice → 35mg
B. Corn → 368 mg
C. Broccoli → 316mg
D. Lima beans → 508 mg

Situation: Nursing theories are vital in advancing frameworks that guide and improve the
delivery of care. Nurse Urielle, the head nurse of Ward X, called for a staff development
meeting.

7. Which nursing theory emphasizes the delivery of nursing care for the whole individual in
order to achieve the physical, emotional, intellectual, social, and spiritual needs of the
patient and the family?
A. The 21 nursing problems → faye abdellah
B. The 14 components of basic nursing care → virginia henderson
C. Human-to-human relationship → joyce travelbee
D. The conservation principles → myra levine

8. Which of the following nursing theorists emphasizes the establishment of a nurse-client


relationship?
A. Jean Watson → theory human caring
B. Hildegard Peplau
C. Josephine Paterson → humanistic theory
D. Martha Rogers → science of unitary human being

9. Central to this nursing theory is promoting behavioral responses which affect health. This
theory is called?
A. Conservation Principles
B. Roy’s Adaptation Model
C. Interpersonal Theory
D. Humanistic Theory

10. What nursing theory explores the assumption of a patient who may be wholly or partially
dependent on the healthcare provider or caregiver?
A. Self-Care Deficit Theory → dorothea orem
B. Conservation Principles
C. Humanistic Theory
D. 21 Nursing Problems

11. The theory of Florence Nightingale focuses on which core principle?


A. A clean and safe environment will facilitate favorable patient outcomes.
B. Mutual trust and respect for other humans are essential to nursing care.
C. Self-care and therapeutic touch are valuable.
D. A good relationship between the patient and the nurse should be dynamic.

Situation
Patient ABC is admitted in hospice care. She is in the terminal phase of ovarian cancer. You
are assigned to take care of Patient ABC and her family. The following questions apply.

12. Which of the following signs of imminent death or active dying is observable in these
patients?
A. Euphoria
B. Uncontrollable pain
C. Clinical depression
D. Inability to swallow → s/t decreased muscle tone

Signs of Impending clinical death


● Loss of muscle tone
● Slowing of circulation
● Changes in respirations
● Sensory impairments

13. As a knowledgeable nurse, you are aware that which of the following choices is a
cardiovascular sign of imminent death?
A. A significant increase in heart rate
B. A rise in blood pressure
C. Increased peripheral circulation
D. Diminished volume of Korotkoff sounds

14. For a patient who is in the terminal stage of illness, the most appropriate lighting
source would be?
A. Softened, sufficient lighting that is enough to distinguish people’s faces.
B. Fluorescent, to prevent harsh and possibly scary shadows.
C. Dim–near darkness–to prevent sensory overload.
D. None of the above.

15. As a nurse who applies collaboration into clinical practice, with whom should you work
with in addressing spiritual needs of the patient?
A. Psychologist
B. A fellow nurse or doctor who is also very religious
C. Hospital chaplain
D. Members of a religious organization

16. With the help of a hospice pastoral care team, you determine the spiritual needs of your
patient. Which of the following information is relevant to take note?
I. The patient’s religious traditions and beliefs
II. Whether the patient acknowledges a god or a significant deity in her life
III. Values, priorities, and cultural beliefs of family members
IV. A spiritual group or community where the patient belongs in
A. i and iv
B. i, ii, and iv
C. i and ii
D. i, ii, iii, and iv

Situation You are caring for a patient diagnosed with metastatic renal carcinoma who is
receiving Interleukin-2 (IL-2) as part of his therapy. The following questions apply.

17. In educating the patient about the purpose of the therapy, which of the following is
correct about Interleukin–2? → stimulate production of lymphocytes, cytokines
A. Forcibly enters malignant cells in their resting phase into mitosis.
B. Increases the patient’s immunologic defenses against tumor cells.
C. Shields the kidney cells from chemotherapeutic adverse effects.
D. Prevents decreased red blood cell, white blood cell, and platelet counts due to
chemotherapy.
18. The patient who is receiving IL-2 complains of symptoms that may indicate an adverse
reaction to the drug. Which of the following symptoms is of utmost priority for the nurse to
report?
A. Generalized body aches
B. Dyspnea
C. Decreased appetite
D. None of the above

19. As a knowledgeable nurse, you should be aware that the patient who is receiving IL-2
therapy may manifest physiologic changes such as?
A. Integumentary changes
B. Skeletal changes
C. Neurologic changes
D. Metabolic changes → anorexia

20. While charting, you note that the patient has a nursing diagnosis of “Risk for injury
related to side effects of chemotherapy”. Which of these options is an appropriate
outcome for this diagnosis? The patient will…
A. Demonstrate productive and adaptive behaviors indicative of problem-solving ability.
B. Demonstrate knowledge of rationale for chemotherapy and treatment plan.
C. Experience toxic effects of chemotherapy.
D. Remain free from infection.

Situation: Nurse Fifi is conducting a qualitative, phenomenological research study on the


stigma of AIDS and its effect on patients with AIDS. Last week, she was able to recruit 6
participants: 3 teenage boys and 3 teenage girls.

21. The most appropriate method to use in collecting data about these participants is?
A. Observation
B. Survey questionnaire
C. Focus group discussion
D. Individual interviews

22. The most appropriate sampling method for this study is?
A. Purposive
B. Stratified
C. Random
D. Snowball → if sensitive ang topic

23. Based on the study design, the findings of the study should be formulated through which
methodological process? → phenomenological
A. Assigning themes and drawing out meanings from the statements of the participants
B. Defining answers of the male and female participants
C. Identifying answers through percentages
D. Effectively summarizing the insights of the participants from one sex, and comparing
them to the insights of participants to another sex, before finally concluding a universal
meaning for both sexes

24. From whom should Nurse Fifi obtain the informed consent for this study?
A. From the six participants
B. From the six participants and their parents
C. From the parents of the participants
D. From the six participants and their available relatives
25. Which of the following is an unrecommended action for Nurse Fifi if she plans to listen to
the tape recordings?
A. Look into the pauses of the participants while speaking.
B. Analyze the voice tone and voice infection.
C. Listen to the audio recordings as soon as possible.
D. Listen to the audio recordings when she feels more motivated to do work.

Situation: In any interaction, communication is a vital tool that facilitates therapeutic


outcomes.

26. In a conversation, nurses should not only be active listeners. They should also validate
what has been said to them by the patient and/or the patient’s significant others. All but one
are the importance of validation:
A. Validation assists in verifying the meaning behind the thought.
B. For cognitively impaired patients, validation helps in extracting important data.
C. Perception affects how a message is interpreted.
D. Eye contact does not necessarily equate to a meaningful message.

27. When a nurse initiates the transaction of thoughts and messages, he employs which
element of communication?
A. Message
B. Initiator
C. Sender
D. Channel

28. If the nurse is encountering a patient who is distressed about her condition, what is the
best means to show effective communication?
A. Sharing
B. Empathizing
C. Sympathizing and analyzing
D. Listening

29. The following are valid reasons for the nurse to continue to work on her communication
skills. Which is the exception?
A. Promulgates threat between her and the patient. → promulgate understanding
B. Decreases incidents of legal issues.
C. Gets a higher evaluation rating on care delivery.
D. Brings change that uplifts the well-being of patients.

30. Nurse Thalia is interacting with her patients in a face-to-face setting upon admission.
Which level of communication is she exhibiting?
A. Verbal
B. Public communication
C. Intrapersonal
D. Interpersonal

Extra personal = human being to non-human being


Intrapresonal = within himself
Interpersonal = between or among tow or more people
Organizational = among organization
Mass communication = among the mass/publis
Situation: To ensure the integrity of data within a healthcare institution, proper
recordkeeping should be employed by the organization to safeguard both employees and
patients.

31. Incident reports should be collected for the day. This should be backed up by due process
and investigation, as scheduled by the Quality Department. Once the investigation is
through, what happens to the incident reports?
A. Summarized monthly and stored in a secure cabinet file.
B. Classified by date for easy access.
C. Completed and stored in an open, easily-accessed cabinet.
D. Put on top of the manager’s desk for easy traceback.

32. Daily charts are collated every three nights across different departments. The night shift
nurse should do the following except:
A. Arrange and bind the charts as they are.
B. See that the charts are complete.
C. Tape and repair torn pages.
D. Ensure the correct order of the charts.

33. Once the patient record reaches the medical records, the assigned employee should?
A. Bind the charts as soon as possible.
B. Check for completeness of the charting of doctors and nurses.
C. Separate the medicolegal charts. → legal aspect
D. Store the charts in their respective shelves.

34. For how long are charts stored in the Medical Records?
A. 1-3 years
B. 5-10 years
C. 3-5 years
D. 1-5 years

35. For how long are medicolegal charts stored?


A. 5 years
B. 8 years
C. 10 years
D. 10 years after the case is closed

Situation: Misconceptions and questions about HIV-AIDS are still rampant globally. Thus,
nurses should take a proactive action on health education for patients and their significant
others. Mimi, 34 years old, married, has been recently diagnosed with the disease.

36. Mimi knows about the mother-to-child transmission of HIV. She asked the nurse when
this transmission happens. The nurse is correct if she answers the following statements,
except? → breastmilk, amniotic fluids, blood
A. In utero
B. During typical contact
C. During breastfeeding
D. During the birthing process

37. Mimi asked the nurse about preventive measures which lower the risk of transmitting
HIV. Nurse Celia tells her that the most effective material for a male condom that can
decrease the transmission of HIV is?
A. Lambskin → small holes
B. Polyurethane → plastic but then easier to break
C. Latex → best protection ever against hiv accdg to CDC
D. Non-latex

38. Mrs. Mimi’s CD4 lymphocyte count was obtained and it was revealed that her results were
below 200 cells/cumm. She asks Nurse Celia what this means. Nurse Celia is correct when
she answers?
A. It may be worrisome but immediate attention is not a requirement.
B. It is slightly below normal so there’s nothing to worry about.
C. The results are still within normal limits.
D. She has Stage 3 HIV-AIDS.

Stage 1: Acute HIV: CD4 count >500 cells.mm3


Stage 2: Chronic HIV: CD4 Count 200-499 cells/mm3
Stage 3: AIDS:CD4 count <200 cells.mm3

39. As an evidence-based nurse, Nurse Celia counsels Mrs. Mimi that the preventative
method of HIV that is not totally realistic is which?
A. HIV testing
B. Behavioral interventions and modifications
C. Promoting linkages to treatment centers
D. Complete abstinence → adolescent – young adults

40. Nurses who are at risk for HIV exposure require post-exposure prophylaxis. In these
options, PEP is described accurately with which exception?
A. The medication must be taken within 72 hours of exposure.
B. 2-3 antiretroviral drugs must be taken, as prescribed.
C. Drugs must be taken for 28 days.
D. Drugs must be taken for at least a week. → every day for 28 days; ideally, started within
72hours after exposure

Situation: Kate, a 28-year-old woman who is married and has two children, recently
underwent a left breast mastectomy. She is deeply worried about how this surgery will
impact her husband and her family.

41. Which of the following statements can help alleviate Kate's worries?
A. "I understand your feelings. My sister underwent a similar surgery half a year ago."
B."Rest assured, I believe your husband and children will be understanding."
C. "It's advantageous that you had the surgery while you were on vacation."
D. "Believe that something greater awaits you from God; keep holding onto hope."

42. What would be the most appropriate response from the nurse to alleviate Mariel's
concern about choking and losing the ability to speak due to the tracheostomy tube?
A. "The tracheostomy tube is temporary and mainly for managing excessive secretions."
B. "Rest assured, choking won't happen because your food doesn't pass through the
tracheostomy tube."
C. "You mentioned feeling anxious about choking and speech difficulties due to your
tracheostomy tube and NGT, is that correct?"
D. "No need to worry; I'll provide you with a pencil and notepad for communication."

43. Which of the following statements made by the client preparing for surgery indicates a
lack of understanding or knowledge?
A. "I must ensure my consent is signed before the surgery."
B. "It's crucial for me to keep my ID bracelet on during the surgery."
C. "I should take Valium to relax before the surgery."
D. "I don't need to follow nothing by mouth guidelines."
44. Leslie, a 36-year-old who recently had a right breast mastectomy, informs you that she
feels nauseated. What should be your primary course of action?
A. Reassure Leslie that nausea can be a common side effect of anesthesia.
B. Position Leslie on her side and prepare an emesis basin. → in case pt vomits → risk for
aspiration
C. Recommend consulting the anesthesiologist and obtaining an antiemetic prescription. →
after ensuring pt safety
D. Assess Leslie's vital signs.

45. While assessing Matet, who recently had a partial gastrectomy, you noticed her blood
pressure decreased from 118/70 mmHg to 90/60 mmHg. You suspect early signs of
hypovolemia and early shock. What should be your initial action?
A. Monitor her heart rate and respiratory rate for a full minute.
B. Examine her surgical incision site and check the nasogastric tube drainage for signs
of bleeding.
C. Position Matet in a shock posture, lying flat on the bed with her legs elevated.
D. Raise the oxygen inhalation rate to 6 liters per minute.

46. Garreth, who recently had an exploratory laparotomy, has been transferred to the
surgical unit. You accompanied Garreth and provided an endorsement to Noah, the nurse in
charge of his care on the surgical floor. Arrange the following interventions in order of
priority:
1. Assess breathing and initiate oxygen inhalation if necessary. → 4th
2. Evaluate the surgical site and examine the wound drainage. → 3rdd
3. Assess the level of consciousness, orientation, and extremity mobility.
4. Connect all drainage tubes to the appropriate drainage system and continuously monitor
them. → 1st
5. Monitor vital signs and take note of skin color, temperature, and the time of the last pain
medication administration. → 2nd
6. Evaluate pain levels, pain characteristics, and the time, type, and route of the last pain
medication administered. → 5th
A. 2, 4, 3, 5, 6, 1
B. 4, 2, 6, 5, 1, 3
C. 4, 5, 2, 3, 6, 1
D. 5, 4, 2, 6, 1, 3

Situation: Susan, a 38-year-old dressmaker, sought emergency medical attention because of


intense abdominal discomfort, accompanied by nausea and vomiting. She displayed
signs of anxiety and irritability. Her vital signs are as follows: Blood pressure: 180/96
mmHg, Pulse rate: 100 bpm, Respiratory rate: 34 cpm.

47. What should be the primary response of the nurse regarding the patient's previous
complaints?
A. Immediately contact the physician
B. Advise the patient to lie down and rest
C. Provide the patient with reassuring words of comfort
D. Prepare the required intravenous fluids

48. Susan has been admitted for a suspected gallbladder issue and has been scheduled for
an ultrasound. What health teaching should you provide before the diagnostic procedure?
A. Fast overnight
B. Consume fruits and coffee before the procedure
C. Drink a glass of water before the test → pelvic utz
49. The physician has informed the patient that she is scheduled for a cholecystectomy with
bile duct exploration. What is the immediate nursing care required when the client has a
T-tube?
A. Examine for bleeding at the tube site → complication
B. Evaluate the color and volume of drainage during each shift
C. Empty the contents of the tube drainage bag once daily
D. Perform tube irrigation with an antiseptic solution

50. What is your primary nursing diagnosis immediately following the surgery?
A. Impaired skin integrity related to altered biliary drainage
B. Acute pain and discomfort about self-care related to knowledge deficit
C. Imbalance nutrition: less than body requirement related to surgery
D. Non-compliance to incision care related to knowledge deficit

51. Which of the following dietary recommendations would be suitable for the nurse to
provide to Susan upon her discharge?
A. Low purine, high protein diet
B. High fat, high carbohydrate diet
C. Low fat, high protein diet
D. High protein, high carbohydrate diet

Situation: Aling Aida, aged 78, experienced a thrombotic stroke three months ago, resulting
in left-sided weakness and difficulty swallowing. During her admission to the medical
ward, she mentioned weight loss, and her daughter added that her mother had not
consumed any food in the last 24 hours.

52. A percutaneous endoscopic gastrostomy (PEG) tube has been recently inserted to
deliver specialized nutritional support to the patient. When administering tube feeding, the
head of the bed should be raised to a minimum of _.
A. 30 degrees → to avoid risk for aspiration
B. 50 degrees
C. 75 degrees
D. 80 degrees

53. Patients receiving enteral feeding typically face a risk of _______.


A. Aspiration
B. Abdominal distention
C.Nausea
D. Infection

54. When providing feedings directly into the stomach, it is crucial to regularly monitor
gastric residual volumes every _____. → 4-6 hrs
A. Every 6-8 hours
B. Twice a day
C.Once a day
D. Every 2 months

55. If the gastric residual volume is less than 500 mL, the nurse should:
A. Promptly alert the doctor
B. Proceed with PEG feeding
C. Wait for an additional hour and reevaluate the residual volume
D. Pause enteral feeding and reevaluate the patient's tolerance
>500 ml = stop and refer to DR → should never be returned
250-500 ml = continue feeding and recheck q4-6 hrs and if >250 ml refer to DR
Never stop feeding unless ordered by Dr
56. Among the common issues faced by patients undergoing tube feeding, diarrhea is a
concern. In such cases, the primary nursing response should include all of the following
except:
A. Investigate potential medications contributing to diarrhea
B. Switch to a high-fiber formula as prescribed
C. Administer the feeding at room temperature
D. Temporarily suspend the formula or water intake

Situation: The nurse admits a 28-year-old male patient who incurred burns while assisting in
extinguishing a fire at a neighbor's residence.

57. The nurse notes that the client has burns that involve the entire surface of both arms
18% and the front of the trunk 18% The nurse calculates the percentage of burned surface
area using the rule of nines, which is:
A. 18%
B. 36%
C. 40%
D. 45%

58. The client is in pain, and the nurse understands that during the initial 24 hours following a
burn injury, pain is managed by providing:
A. Narcotics injected into non-burned tissues via intramuscular route
B. Intravenous narcotic medications
C. Tepid baths and oral morphine
D. Liquid narcotics delivered through a nasogastric tube

59. The nurse examines the client's stool for hidden blood to evaluate the presence of
which of the following medical conditions?
A. Intestinal ileus
B. Stress ulcers → curling’s ulcer
C. Gastric irritation
D. Bleeding related to bowel distention

60. The doctor conducts enzymatic debridement on the client. The nurse will then evaluate
the client for:
A. Increased fluid loss → unrelated
B. Allergic reactions → ointments, gel
C. Serum electrolyte imbalance
D. Bleeding from the burned area

61. The nurse administers a tetanus vaccination to a client when the client has ______:
A. Open wounds with a significant amount of embedded debris
B. Third-degree burns
C. Second-degree burns with open blisters
D. Received a vaccination within the past six years → up to 10 yrs

Situation: Nurse Jeline serves as the charge nurse in a trauma unit. She employs
management principles to enhance the quality of patient care, with a particular focus on
optimizing resource utilization and managing the patient's environment effectively.

62. Nurse Jeline recognizes that environmental factors have a significant impact on patient
care. The nurse's primary priority for the patient is ________:
A. Ensuring client safety
B. Increasing staffing levels
C. Maintaining confidentiality
D. Ensuring an ample supply of pain relief medications

63. Nurse Jeline collaborates with other healthcare professionals. She recognizes that an
interdisciplinary care plan encompasses:
A. All nursing staff members who provide input into the care plan
B. Both the physicians and nurses involved in patient care
C. The client's explicit preferences and advance directives
D. Contributions from all disciplines involved in the patient's care

64. Nurse Jeline recognizes that implementing quality measures in the unit leads to cost
savings in patient care. The following are examples of inefficient resource utilization,
except: (+)
A. The time nurses spend searching for missing supplies or laboratory results
B. Delays in client discharge caused by a lack of coordination among staff
C. The time nurses invest in documenting client treatments, medications, and outcomes
D. The expenses associated with overtime pay due to unfilled positions

65. Nurse Jeline creates a workload budget for the team. What is the objective of a
workload budget? → amount of work that is expected to do; budget: financial plan
A. Quantity of work generated by a department, measured in units of service used to
calculate expenditure budgets
B. The mean number of patients attended to in a day over a specified time frame
C. Gathers statistics on present activity, with a focus on the number of units of service
provided

66. Nurse Jeline implements a quality management program known as Lean Six Sigma,
which centers on enhancing process efficiency and eliminating inefficiencies. The following
statements accurately reflect resource management, except:
A. Healthcare professionals aim to minimize errors and enhance client safety
B. Effective planning is a crucial element of resource management
C. Typically, augmenting nursing staff leads to improved patient outcomes
D. Waste occurs when a unit allocates fewer resources than needed → more

Situation: An 8-year-old girl, accompanied by her parents, visits the outpatient department
due to complaints of pain in her ankles, feet, and knees. The initial diagnosis upon admission
is Juvenile Rheumatoid Arthritis (JRA).

67. The parents express their concerns to the nurse regarding their child's disease outcome.
Based on her knowledge of the condition, the nurse should clarify that:
A. Typically, as the child grows into adulthood, the disease tends to advance and may lead to
debilitating rheumatoid arthritis
B. In the majority of cases involving children, the disease enters a state of permanent
remission
C. While many children experience extended periods of remission, they may also suffer from
severe deformities and a decline in their physical function
D. The majority of affected children typically achieve full recovery within a few years

68. Which nursing diagnosis formulated by the nurse is suitable for a young child diagnosed
with JRA?
A. Impaired Physical Mobility
B. Impaired Swallowing
C. Ineffective Airway Clearance
D. Total Incontinence
69. The mother inquires of the nurse about the appropriate course of action to take upon
noticing her child's withdrawn behavior. The nurse's recommendation would be to:
A. Offer support and understanding to her child
B. Facilitate interactions with other children who have JRA
C. Allocate additional time to spend with the child while less time for her other children
D. Arrange a meeting with the school guidance counselor for her child

70. Which of the statements below accurately describe Juvenile Rheumatoid Arthritis (JRA)?
1. JRA is the predominant chronic condition among individuals under 16 years old
2. The disease typically begins around the age of 8 and tends to peak during puberty → 1-3
yrs old
3. Initial symptoms are often referred to as growing pains
4. Methotrexate (MTX) is generally more effective in treating JRA in children compared
to adults
A. 1 & 2
B. 1, 3 & 4
C. 1, 2, 3 & 4
D. 2 & 3

Situation: The nurse analyzes the results of a recent study, which indicated that while
individuals with depression may face an increased risk of heart failure, those who have
cardiovascular disease are more likely to experience symptoms of depression.

71. The study also uncovers that major depression is most commonly found among which of
the following groups:
A. Males
B. Females
C. Individuals under the age of 60
D. Individuals aged 60 and above

72. The nurse comprehends that individuals with chronic heart failure experience the most
significant mood swings when they:
A. Simultaneously suffer from other cardiovascular conditions
B. Harbor uncertainties regarding the progression of their illness
C. Encounter challenges in maintaining their employment
D. Previously enjoyed highly active lifestyles

73. A term referring to a chronic state of depression wherein the individual encounters fewer
than five depressive symptoms persisting for a duration of two years is called _.
A. Minor depression
B. Major depression
C. Dysthymia
D. Depression psychosis

74. The research examined various factors, including physiological interventions for
individuals experiencing both heart failure and depression, in addition to psychological and
psychotherapeutic approaches. The findings revealed that psychosocial and
psychotherapeutic interventions:
A. May not be preferred by the majority of clients
B. Require a longer duration to achieve effectiveness
C. Demonstrate reduced effectiveness
D. Tend to exacerbate dyspnea
Situation: The nurse manager responsible for the medical and surgical units conducts a
reorientation program for the staff nurses and other healthcare team members regarding
the proper procedures for reporting and documenting any incidents that might occur within
the units.

75. The nurse manager informs the staff that the reasons for writing incident reports include
all of the following except for:
A. Enhancing the quality of nursing services
B. Providing an objective account of unexpected events
C. Taking disciplinary action
D. Documenting and conducting follow-up procedures

Situation: The nurse provides care to a cohort of patients who have different forms of
cancer and are undergoing cancer treatment and taking prescribed medications.

76. A drug is administered to a client as part of a Phase 1 clinical trial. The nurse presumes
that the cancer:
A. Is not covered by the client's insurance, along with its treatment
B. Is unresponsive to established cancer treatments
C. Is deviating from the anticipated disease progression
D. Is small in size and less severe

Clinical trial
Phase I - Safety and proper dose - 15-50 pt
Phase II - effectiveness and side effects - less than 100 pt
Phase III - compare new treatment to existing tx - hundreds of people
Phase IV - tx approved and available - thousands of people

77. A Phase II clinical trial medication is administered to a patient diagnosed with lung
cancer. Which of the following represents a suitable evaluation of the medication by the
course?
A. Identification of the highest tolerated dosage
B. Characterization of the nature and extent of adverse reactions
C. Appraisal of the overall efficacy of the medication
D. Description of how the medication measures up against established treatment protocols

78. The nurse responsible for a patient with an implanted radiation source should minimize
their own radiation exposure by implementing which of the subsequent methods?
A. Restricting the duration of proximity to the patient to thirty minutes per eight-hour shift
B. Utilizing a radiation meter or film badge to monitor radiation exposure
C. Maintaining a six-foot distance from the patient except when providing hands-on care
D. Donning a lead-shielded apron upon entering the patient's room → reduce
self-exposure

79. For a 30-year-old patient with an implanted radiation source, which of the following
nursing tasks takes precedence?
A. Mitigate radiation-related skin issues
B. Evaluate the client's response to diagnosis and treatment
C. Encourage regular physical activity during confinement
D. Ensure the client and others are protected from unnecessary radiation exposure →
safety

80. The nurse tending to a patient with a temporary radioactive cervical implant should
refrain from which of the following actions?
A. Leaving the client's bedding in a laundry hamper in the hospital corridor → IV/IM
unsealed
B. Spending an hour conversing with the client by the bedside → 30 mins per shift lang
C. Flushing the toilet only once after emptying the client's bedpan
D. Administering an alcohol-based mouthwash for the client's oral hygiene

Situation: A male child, aged 10, has been admitted to the surgical department's pediatric
unit at Hospital X due to complaints of pain in the lower right abdomen. The preliminary
diagnosis suggests acute appendicitis.

81. Which of the subsequent signs and symptoms is most likely to manifest in a client
suffering from appendicitis?
A. Convulsions, fluctuations in weight
B. Elevated body temperature, feelings of nausea, and vomiting
C. Sore throat, bone and joint aches
D. Pruritus and edema around the eyes and ankles

82. The nurse conducts a physical examination. What assessment outcomes would the nurse
anticipate in a child with appendicitis?
1. Sore throat, bone and joint aches
2. Diarrhea or sudden constipation
3. Focal pain in the lower left region
4. Tensing of the abdominal wall muscles
A. 1 & 3
B. 1, 3 & 4
C. 1, 2, 3 and 4
D. 2 & 4

83. During the assessment, when the child mentions that the pain has disappeared and their
abdomen no longer hurts, the nurse might suspect that:
A. The appendix has ruptured
B. The child is anxious about undergoing surgery
C. The child seeks additional attention
D. The child encounters challenges in effectively communicating their pain

84. The client's diagnosis has been verified, and an appendectomy is scheduled. Preoperative
nursing care involves which of the following:
A. Applying warm compresses to the child's abdomen to alleviate discomfort
B. Administering a prescribed laxative
C. Evaluating bowel activity through the assessment of abdominal distension and
listening to bowel sounds
D. Positioning the child on their left side to aid in localizing and preventing the spread of
infection

85. The nurse readies the child for surgery. Which of the following nursing actions should the
nurse undertake?
A. Providing clear fluids → NPO
B. Applying warmth to the abdominal area → Perforation
C. Administering an enema as prescribed → inc IAP → perforation
D. Ensuring that the child remains in complete bed rest

Situation: Engaging in research is an essential pursuit that nurses must undertake to make
meaningful contributions to the field of nursing science.
86. What type of design is employed when a nurse researcher collects data at multiple time
points over an extended period?
A. Time-related
B. Time-sequenced
C. Cross-sectional → single/specific point in time
D. Longitudinal

87. Which type of research design is indicated when a research study includes interventions
and binding?
A. Descriptive
B. Non-descriptive
C. Experimental
D. Phenomenological

88. Which of the following statements is the least descriptive of a qualitative research
design?
A. It is flexible and elastic
B. Gather one data from collection strategy → multiple
C. Strives for an understanding of the whole strategy
D. Researchers become involved

89. Qualitative researchers should select participants who align most effectively with the
study's objectives. Who among the following individuals is the most suitable choice?
A. Cooperative community members
B. Those who are readily available and thus convenient for the researcher
C. Individuals capable of articulating and reflecting on their experiences with the
phenomenon
D. Individuals referred by friends

90. In research, a "full understanding" should be grasped by the nurse researcher as:
A. Ensuring that the participants' safety is not compromised
B. Clearly elucidating the study, encompassing risks and benefits
C. Upholding the participants' right to make voluntary decisions
D. Avoiding any exploitation of the information shared by the participants

Situation: Effective record management is crucial within any healthcare facility. It is the
nurse's responsibility to ensure that the task is carried out with utmost care and attention.

91. Nurse Pauline, while working in the medical unit, is instructed to use abbreviations that
adhere to the following criteria:
A. Automatically utilized to save valuable time
B. Minimized in usage across all units
C. Part of the hospital's approved standard list
D. Avoided altogether to prevent potential misinterpretations

92. A mistake in record-keeping can occur due to illegible handwriting. In such a situation,
what should the nurse do?
A. Seek assistance from a senior nurse to clarify the order
B. Consult the resident-on-duty at the nurses' station for interpretation
C. Contact the physician who issued the order
D. Report the lapse to the Quality Assurance Committee
93. When a nurse makes an error in the progress notes, the most appropriate course of
action is:
A. Cross out the error repeatedly to ensure it becomes unreadable and then sign
B. Delete the erroneous phrase or sentence, make the correction above it, and sign
C. Draw a line through the sentence, make the correction above it
D. Erase the error using a rubber eraser and then sign

94. Which of the following statements is incorrect concerning record-keeping?


A. Neglecting it could serve as evidence of professional misconduct
B. It is an optional task to be completed when circumstances permit
C. It is a professional practice tool that aids in delivering quality care
D. It is a fundamental part of a nurse's professional duties

95. The QA nurse routinely performs an audit of medical records. The primary objective of
conducting an audit within a healthcare facility is to:
A. Detect errors made by healthcare personnel
B. Identify areas for enhancement
C. Verify compliance with established standards
D. Facilitate risk management initiatives

Situation: Prior to Ruru's surgery, health education sessions are arranged for him and his
family. The health education plan focuses on achieving weight gain through a diet rich in
protein and calories, and it is being prepared by the nurse.

96. The nurse ensures that which of the following groups should be present and actively
participate in the educational program?
A. Patient, family members, and significant others
B. Patient, student nurses, and interns
C. Head nurse and family members
D. New staff nurses and nursing aides from the unit

97. In the education plan, the nurse must incorporate which of the following components?
Select all that apply.
I. Objectives
II. Content and allotment
III. Teaching and learning resources
IV. Evaluation criteria
A. I, II, and III
B. I, III, and IV
C. II, III, and IV
D. I, II, III, and IV

98. To facilitate a simpler and more comprehensible implementation of the plan, the nurse
conveys it using which of the following?
A. A lecturer
B. A colleague
C. Colorful illustrations
D. Printed content on cartolina → pictures, words

99. Prior to finalizing and implementing the staff nurse's education plan, it is most advisable
to have it reviewed by the:
A. Head Nurse
B. Nurse Supervisor
C. Chief Nurse
D. Medical Director
100. Throughout the implementation process, the nurse should ensure a to facilitate better
understanding of the teachings.
A. Suitable time and location
B. Lecture beginning at 11:00 AM
C. Novice as a contributor
D. Serious lecturer

You might also like